Pharmacology- Renal and Cardiovascular (UWORLD)

अब Quizwiz के साथ अपने होमवर्क और परीक्षाओं को एस करें!

66 years man -Lightheadedness, recent fall (especially in morning when he gets up from bed) -Medications for hypertension, stable angina, gout, and benign prostatic hyperplasia -Osteoarthritis= occasional NSAIDs -BP drops on standing -Blockade of which receptor produces these symptoms? a)Alpha-1 adrenergic b)Alpha-2 adrenergic c)Beta-1 adrenergic d)Beta-2 adrenergic e)D1-dopaminergic

answer: Alpha-1 adrenergic Patient= orthostatic hypotension (standing causes pooling of blood into veins below heart which initiates): -Decreased venous return to heart -Decreased ventricular filling and subsequent drop in CO -Drop in BP that evokes compensatory baroreceptor reflex -Increased sympathetic tone that increases peripheral vascular resistance (alpha-1) as well as heart rate and myocardial contractility (beta-1) Alpha-1 antagonists (terazosin, doxazosin)= used for symptomatic relief in benign prostate hyperplasia= associated with orthostatic hypotension - Other etiologies of orthostatic hypotension: volume depletion (hypovolemia due to hemorrhage, hyperglycemia or diuretics); autonomic dysfunction (parkinson, diabetes) option B= blockade= increased release of NE from terminals= increased BP and HR option C= stimulates= increased HR, contractility and conduction; blockade= opposite option D= stimulation= bronchodilation, vasodilation and uterine relaxation; blockade= opposite option E= in walls of renal, splanchnic and mesenteric blood vessels; stimulation= vasodilation and increased blood flow; blockade= opposite

-New drug developed to treat cardiac arrhythmias -Drug= high affinity for activated and inactivated sodium channels but relatively little affinity for resting sodium channels -Experiment to maintain sodium channels in the inactivated state for a prolonged period, during which the new drug is introduced -After allowing the sodium channels to return to resting state, the speed of the drug's dissociation from sodium channels is recorded and shown (compared to quinidine) -New drug most similar to? (learn don't answer) a)Disopyramide b)Flecainide c)Lidocaine d)Propranolol e)Sotalol f)Verapamil

-Class 1 antiarrhythmics= block voltage gated Na+ channels -Bind channel during open and/or inactivated states, dissociation occurs during resting state= state-dependent binding (more depolarized tissues are more susceptible to blockage= use-dependence= the faster the heart rate, the more time the Na+ channels spend in open and inactivated state and more gets blocked -Class IB (e.g., lidocaine)= weakest binding= rapid dissociation (little use dependence)= more selective for ischemic myocardium because the elevated resting membrane potential delays Na+ channel transition from the inactivated to the resting state= increased drug binding= useful for treating ischemia-induced ventricular arrhythmias -Class IC (e.g., propafenone, flecainide) have strongest binding (choice B)= slowest dissociation= greatest blockage at higher heart rates (high use dependence)= useful in terminating tachyarrhythmias (BUT they can cause excessive delay in conduction speed= promote arrhythmias) option A= quinidine and procainamide too; class IA= intermediate dissociation speed option D= class II; decreased chronotropy and inotropy option E= class III; amiodarone and Dofetilide too; block potassium channels and prolong phase 3 option F= class IV; slow SA conduction and prolong AV conduction pause

60 years man -Exertional angina -Treatment: metoprolol, isosorbide dinitrate, aspirin -Isosorbide dinitrate: early in morning, and afternoon, but not evening -Pattern of dosing to decrease? a)Pharmacokinetic drug interaction b)Pharmacodynamic drug antagonism c)Effect potentiation d)Tolerance development e)Withdrawal symptoms f)Drug noncompliance

answer: tolerance development -Around the clock nitrate administration (in any form)= tolerance development to nitrates -Prevention: nitrate-free interval everyday in patients using daily long-acting nitrates (usually at night when cardiac work is the least) -Theory of mechanism: decreased vascular sensitivity to nitrates and increased sensitivity to endogenous vasoconstricting agents

52 years woman -History: locally advanced breast cancer -Undergoing trastuzumab-based chemotherapy (no radiation therapy) -Mild fatigue, transthoracic echocardiography= LV ejection fraction of 40% -Cardiac assessment before chemotherapy= normal -Pathology? a)Abnormal protein accumulation in the extracellular space b)Focal ischemic cardiomyocyte necrosis c)Myocardial tumor cell infiltration d)Patchy cardiomyocyte necrosis with diffuse myocardial fibrosis e)Reduced cardiomyocyte contractility with no fibrosis

Answer: Reduced cardiomyocyte contractility with no fibrosis -Trastuzumab is a monoclonal antibody used in the treatment of breast cancer caused by tumor cells that overexpress human epidermal growth factor receptor-2 (HER2) -By binding to HER2, trastuzumab blocks downstream signaling that promotes cellular proliferation and thereby encourages malignant cell apoptosis -Adverse effects: cardiotoxicity (likely because HER2 signaling plays a role in minimizing oxidative stress on cardiomyocytes and preserving cardiomyocyte function)= manifests as a decrease in myocardial contractility (myocardial stunning) without cardiomyocyte destruction or myocardial fibrosis -Manifestation: asymptomatic decline in LV ejection fraction; however overt heart failure can occur -Unlike, cardiotoxicity that occurs that occurs with anthracyclines (e.g., doxorubicin; causes dilated cardiomyopathy), trastuzumab-induced cardiotoxicity is not related to the cumulative chemotherapy dose and is often reversible with discontinuation of therapy option A= amyloidosis, light chain (AL) and transthyretin (ATTR) subtypes; manifests initially as restrictive cardiomyopathy with impaired diastolic function and preserved ejection fraction option B= leads to LV systolic dysfunction; seen in patients with prior MI due to CAD option C= breast cancer is one of the most common malignancies that metastasize to the heart; typically causing pericardial effusion that may progress to tamponade; tamponade predominantly affects diastolic filling of the right sided heart option D= anthracycline-induced cardiotoxicity; oxidative damage of cellular membranes= cardiomyocyte necrosis and death with eventual fibrosis

78 years woman -Altered mental status -Medical history: pyelonephritis, type 2 diabetes, coronary artery disease, heart failure with reduced ejection fraction -Low BP, high pulse and RR; O2 saturation= 90% -Exam: bilateral crackles -Initiation of norepinephrine infusion= BP increases, and HR slightly decreases -Effect of norepinephrine on HR due to? a)Alpha-1 receptors in blood vessels b)Alpha-2 receptors in the medulla c)Beta-1 receptors in the heart d)Beta-2 receptors in blood vessels e)Dopamine receptors in blood vessels

answer: Alpha-1 receptors in blood vessels -NE= commonly used in patients with shock -Directly and potently stimulates alpha-1 receptors= vasoconstriction (increases SVR and BP) -Increase in SVR= indirect reflex (baroreceptor reflex) -Increased BP= increased firing of aortic and carotid baroreceptors= increased PSNS output and decreases SNS output= decreases HR and contractility (oppose direct effects on beta-1) -Alpha-1 action stronger than beta-1= HR either unchanged or slightly decreased option B= clonidine (agonist); decreases SNS= vasodilation, decreased HR, and reduced BP option C= NE stimulates beta-1= increases HR, contractility and CO option D= NE= mild vasodilation option E= dopamine; renal vasodilation

56 years man -24 hours of burning substernal chest pain (most intense 6 hours ago and then started to subside) -History: diabetes -ECG= ST elevations in leads V3-V5 -Cardiac catheterization lab= total occlusion of LAD artery -Successful intervention performed -Next day= normal sized LV with no evidence of hypertrophy, there is apical and anterior akinesia with reduced LV ejection fraction of 38% -Long-term effect of ACE inhibitor therapy in this patient? a)Attenuation of left ventricular chamber dilation b)Enhanced collagen deposition in the apical myocardium c)Enhanced collagen deposition in the peri-infarct myocardium d)Enhanced left ventricular hypertrophy in the basal segments e)Increase in left ventricular stroke work

answer: Attenuation of left ventricular chamber dilation -Reduced LV ejection fraction due to acute MI -Echo= akinesia of apical and anterior segments= MI in these locations -Following MI (months)= affected myocardium undergoes fibroblastic proliferation and fibrosis to repair the necrotic region (i.e., initial remodeling phase); disproportionate thinning of the infarcted area can lead to ballooning of the affected segments and marked LV cavity enlargement -Although LV cavity enlargement is initially beneficial as it acts as compensatory mechanism for LV volume overload, progressive cavity enlargement (e.g., eccentric hypertrophy) leads to overwhelming wall stress and further worsening of LV contractile dysfunction (with progressive dilation, the LV assumes a spherical shape) -Much of the remodeling is drive by AT II= ACEIs reduce the deleterious remodeling that takes place following MI, minimizing LV dilation and helping preserve LV function

54 years man -New-onset muscle cramps -History: hypertension -Medication: hydrochlorothiazide (4 weeks ago) -High BP; triamterene was added to current therapy -Mechanism of action of added drug? a)Blocking renal tubular epithelial sodium channels b)Decreased aldosterone production c)Increased intestinal absorption of potassium d)Inhibiting aldosterone receptor activity in renal collecting duct e)Inhibiting Na+/K+ ATPase activity in skeletal muscle

answer: Blocking renal tubular epithelial sodium channels -Thiazide diuretics= inhibit Na/Cl cotransporters in DCT -Both thiazides and loop diuretics induce volume loss with subsequent activation of RAAS -Aldosterone increases renal sodium resorption in exchange for potassium by upregulation the synthesis and activity of à 1) epithelial sodium channels (ENaC) (apical membrane) 2) Na/K ATPase (basal membrane) -Increased aldosterone= hypokalemia= muscle cramps, weakness, and when severe= arrhythmias Potassium sparing diuretics may be added to prevent hypokalemia -Triamterene and amiloride: inhibit epithelial sodium channel (ENaC); inhibits sodium absorption and reduces electrical gradient that drives potassium secretion -Spironolactone and eplerenone: aldosterone antagonists; decreased formation of ENaC and Na/K ATPase (choice D) option B= ACEIs and ARBs option C= potassium supplements option E= digoxin; antiarrhythmic medication (atrial fibrillation); inhibits cardiac and skeletal muscle pumps= can cause hyperkalemia

65 years man -T2D; decreased sexual performance (6 months) -Libido, muscle strength and energy= normal; but erections are inconsistent, weak, and inadequate for duration of sexual activity -History= hypertension -Treated with sildenafil= symptoms improve -Intracellular signaling most similar to? a)1,25-dihydroxycholecalciferol b)Brain natriuretic peptide c)Gamma-aminobutyric acid d)Insulin e)Interleukin-2 f)Platelet-derived growth factor

answer: Brain natriuretic peptide -Atrial and brain natriuretic peptides (ANP and BNP)= bind ANP receptors (linked to guanylyl cyclase)= converts guanosine 5'-triphosphate to cyclic guanosine 3'-5'-monophosphate (cGMP)= acts on cGMP dependent protein kinase= relaxation of smooth muscle and vasodilation -Penile erection is initiated by release of Ach and NO= rise in intracellular cGMP and vasodilation of arteries supplying corpus cavernosum -Sildenafil is PDE-5 inhibitor= decreases degradation of cGMP -PDE-5 inhibitors + nitrates= accumulation of cGMP= severe hypotension option A= vitamin D; interacts with receptors in nucleus option C= ligand gated ion channels or metabotropic receptors option D and F= tyrosine kinase receptors option E= many receptors; pathways= JAK-STAT, MAPK, PIP3 kinase

64 years man -Exertional chest pain (last 6 months) -Smoker (lifetime); type 2 diabetes and peripheral artery disease -Stress test= substernal chest pain on moderate exertion (relived by rest) -Low-dose aspirin therapy= shortness of breath and wheezing -Best alternative therapy? a)Apixaban b)Cilostazol c)Clopidogrel d)Enoxaparin e)Eptifibatide f)Naproxen g)Warfarin

answer: Clopidogrel -disease: stable angina due to underlying CAD -Stable angina= start aspirin to decrease risk of cardiovascular events Aspirin= impairs prostaglandin synthesis by irreversibly inhibiting COX -Inhibits COX-1 (low doses)= prevents synthesis of thromboxane A2 (potent stimulator of platelet aggregation and vasoconstriction)= reduce risk of occlusive thrombus formation and MI -Some can't take aspirin= allergy (e.g., urticaria, angioedema, anaphylaxis) or preexisting respiratory symptoms (e.g., rhinitis, asthma) Alternate antiplatelet therapy= Clopidogrel (irreversibly blocks the P2Y12 component of ADP receptors on platelet surface and prevents aggregation) -It is as effective as aspirin for prevention of cardiovascular events option A= direct Factor Xa inhibitor; prevent platelet activation and fibrin clot formation; used for prevention and treatment of thromboembolic events, not CAD option B= phosphodiesterase inhibitor; used in peripheral artery disease (i.e., claudication) option C= and dalteparin= LWMH; indirect thrombin inhibitors; bind antithrombin and convert it from a slow to rapid inactivator of thrombin and factor Xa; used in patients with acute coronary syndrome (unstable angina or MI), but no role in stable angina option D= platelet glycoprotein IIb/IIIa inhibitor; inhibits final common pathway of platelet aggregation; occasionally used in acute coronary syndrome but not stable CAD option F= NSAID; used for pain management in patients with osteoarthritis; use of nonselective or selective COX-2 inhibitors increases risk of cardiovascular events option G= (inhibits vitamin K epoxide reductase; required for synthesis of vitamin K; decrease in factors II, VII, IX, and X); used for prevention and treatment of thromboembolic events, not CAD

55 years man -Recurrent episodes of retrosternal chest pain (during physical activity) -Sublingual tablet and reports rapid relief of pain -Hemodynamic changes of drug? a)Decrease in arteriolar resistance b)Decrease in heart rate c)Decrease in left ventricular end-diastolic volume d)Increase in coronary blood flow e)Increase in coronary perfusion pressure

answer: Decrease in left ventricular end-diastolic volume -disease: chronic stable angina (relieved by rest or sublingual nitroglycerine) Nitrates exert effect by direct vascular SMC relaxation: -Vasodilation of peripheral veins and arteries, predominantly venodilation -Decreased left ventricular wall stress due to reduced preload (decreased LV end-diastolic volume and pressure) -Modest reduction in afterload due to systemic arterial vasodilation -Mild coronary artery dilation and reduction of coronary vasospasm Results in decreased myocardial oxygen demand, leading to improvement of angina symptoms option A and D= nitrates dilate large epicardial arteries more than smaller resistance vessels (i.e., arterioles); furthermore, coronary arterioles are maximally dilated in stenosis to maintain resting blood flow; limiting ability of drugs option B= nitrates can increase HR as reflex response to reduced BP; beta blockers and Nondihydropyridine CCBs reduced HR= antianginal effect option E= CPP= pressure gradient that drives coronary perfusion during diastole= end-diastolic aortic pressure- LV EDP; nitrates may slightly reduce coronary perfusion pressure as they slightly lower systemic arterial pressure

-Clinical trial -Drug is pure adrenergic agonist without significant antagonistic properties -Infusion of drug: initially increases SBP and DBP; then after some time decreases HR -Administration of drug in healthy individuals would cause? a)Decreased atrioventricular node conduction velocity b)Decreased cardiac afterload c)Decreased pulmonary capillary wedge pressure d)Decreased venous tone e)Increased renal perfusion

answer: Decreased atrioventricular node conduction velocity -Increase in BP with reflex bradycardia (delayed)= typical effects of selective alpha-1 agonists (e.g., phenylephrine, methoxamine) -Alpha-1 agonists= vasoconstriction (raise BP)= reflex increase in vagal influence (carotid and aortic arch)= slow down SA and AV node conduction and decrease contractility option B= would be increased with pressure option C= would be increased due to increased in afterload and venous tone option D= increased venous tone option E= vasoconstriction= reduced perfusion

34 years woman -Recurrent throbbing headaches, nausea and photophobia (several times a month) -High BP; prescribed metoprolol (expected to decrease frequency of headaches and lower BP) -Mechanism on BP effect? a)Decreased level of circulating catecholamines b)Decreased level of circulating renin c)Increased atrioventricular nodal conduction rate d)Increased renal sodium and water retention e)Long-term increase in peripheral vascular resistance

answer: Decreased level of circulating renin -Beta blockers are useful for treating hypertension with comorbid conditions (e.g., migraine, essential tremor, angina pectoris, prior MI, atrial fibrillation) -Beta blockers lower BP by= 1) reducing myocardial contractility and HR 2) decreasing renin release by kidney -Renin release= mediated by beta-1 receptors on juxtaglomerular cells= beta blockers inhibit renin release option A= block actions of endogenous catecholamines on adrenergic beta receptors option C= depress conduction through AV= varying degrees of AV block option D= beta blockers reduce this option E= no change or modest decreases in peripheral vascular resistant due to effects of RAAS inhibition

54 years man -Nonischemic cardiomyopathy -Hospitalized frequently= acute decompensated heart failure -Dyspnea on mild exertion -History: hypertension -Exam: elevated jugular venous pressure, bibasilar lung crackles and lower extremity edema -3 weeks= digoxin therapy= symptomatic improvement -Initial cellular event triggering response? a)Decreased sodium efflux from myocardial cells b)Increased intracellular calcium concentration c)Increased intracellular cyclic AMP concentration d)Increased potassium influx into myocardial cells e)Troponin sensitization to calcium

answer: Decreased sodium efflux from myocardial cells -Digoxin is a positive inotropic agent= symptomatic relief in patients with acute decompensated heart failure due to LV systolic dysfunction -Also, increases parasympathetic tone and slow conduction through AV node, which improves cardiac function in patients with rapid ventricular rate -Inhibits Na/K ATPase= decreased sodium efflux (increase in intracellular Na)= Na/Ca exchanger dysfunction= decrease in calcium efflux= stimulates binding of calcium to troponin C and subsequent actin-myosin cross bridge formation= improved myocyte contractility option B= secondary effect option C= stimulated by beta agonists and inhibited by Ach; cAMP increases conductance of calcium channels in SER= more calcium to bind troponin C option D= inhibition of Na/K ATPase= decreased potassium efflux option E= digoxin doesn't do that

65 years man -Central chest pain (past 2 hours) -Description: Very severe pain, and going back all the way to my spine -History: hypertension, nonadherent to medications, smoker (20 years) -BP (195/101) and pulse (93, regular) -ECG: LVH; chest X-ray= widened mediastinum -Treated with IV esmolol -Purpose of current therapy? a)Decreasing left ventricular contraction velocity b)Decreased myocardial oxygen demand c)Decreasing peripheral vascular resistance d)Increasing left ventricular filling pressure e)Increasing left ventricular stroke volume

answer: Decreasing left ventricular contraction velocity -Severe chest pain + radiating to back + widened mediastinum= acute aortic dissection -Uncontrolled hypertension (nonadherence to medication, LVH)= strongest risk factor for aortic dissection -With each LV contraction= ejected SV rapidly increases aortic pressure and generates shear stress on aortic wall -Medical therapy aim= reducing aortic wall shear stress to limit extension of dissection= anti-impulse therapy to decrease rate of change in aortic blood pressure per time Esmolol is a selective beta-1 receptor blocker that is given by IV infusion (first-line therapy for acute aortic dissection, as it has short half-life (allowing for easy dose titration) and reduces shear stress in 2 ways: -Negative inotropy decreases LV contraction velocity= reduces SV (opposite of option E)= decrease rise in BP with each contraction -Negative chronotropy decreases heart rate, subjecting aortic wall to fewer LV contractions per minute After shear stress is minimized= decrease PVR (esmolol has little effect on PVR= choice C) to further reduce BP= vasodilators (e.g., nitroprusside, nicardipine) may be added for this purpose Never give vasodilators before esmolol= vasodilation= reflex tachycardia (increase aortic wall shear stress) option B= beta blockers; major goal of medical therapy for treating myocardial ischemia, reducing aortic wall shear stress is much more important in aortic dissection option D= reduced LV contractility and HR may slightly increase LV filling pressure (due to less blood being ejected with each contraction and increased LV filling time), minimal consequence on aortic dissection

-New antiarrhythmic drug (drug A) tested with measuring effect on flow of ions responsible for AP in cardiac muscle cells -results: prolongation of K efflux in repolarization phase drug A is most similar to? a)Adenosine b)Digoxin c)Dofetilide d)Esmolol e)Flecainide f)Lidocaine g)Quinidine h)Verapamil

answer: Dofetilide -Phase 0 (rapid depolarization)= Sodium current in= class I antiarrhythmics -Phase 1 (early repolarization)= transient outward K current -Phase 2 (plateau)= balance between inward Ca and outward K -Phase 3 (late repolarization)= opening of delayed rectifier potassium channels= class III antiarrhythmics -Phase 4 (resting)= normal RMP, outward potassium current through potassium leak channels; Na/K ATPase mediates gradients of Na and K to maintain it -Decrease in outward potassium flow= class III= amiodarone, sotalol, Dofetilide (block potassium channels) option A= stimulates A1 receptors on surface of cardiac cells, activating potassium channels and increasing potassium conductance, which causes membrane potential to remain negative for longer period= transient slowing of sinus rate and increased AV nodal conduction delay option B= inhibits Na/K ATPase= increased intracellular sodium= increased intracellular calcium= improved contractility option D= rapid-acting, short-duration beta blocker; class II; slows rate of discharge of sinus or ectopic pacemakers and increases refractory period of AV node option E= Class IC; inhibit sodium channels option F= Class IB; inhibit sodium channels option G= Class IA; inhibit sodium channels option H= CCBs; inhibit calcium channels= less calcium= slow sinus rate, prolong conduction via AV node; depress contractility

64 years man -Follow-up of hypertension; last 2 visits= BP high -Patient treated with lisinopril (5 years); other medications always changing due to adverse effects -History: T2D, CAD, stable angina, increased albuminuria -BP is high; high BMI -Exam: trace bilateral lower extremity edema; hyperkalemia -Medication that should be avoided? a)Amlodipine b)Bumetanide c)Diltiazem d)Eplerenone e)Furosemide f)Metoprolol

answer: Eplerenone -Treated with ACEI= decreased aldosterone secretion= increased sodium excretion and potassium retention= hyperkalemia (risk is greatest in patients with renal insufficiency and patients taking potassium sparing diuretics or medications that decrease GFR like NSAIDs) -Mineralocorticoid receptor blockers (e.g., spironolactone, eplerenone), ARBs, ENaC blockers (e.g., amiloride, triamterene) are other antihypertensives that increase K levels= concurrent use with ACEIs= severe hyperkalemia option A and C= CCBs may cause lower extremity edema option B and E= loop diuretic= hypokalemia option F= hyperkalemia; metoprolol is Cardioselective= blocks beta-1 only

71 years woman -Progressive dyspnea and orthopnea (last week) -History: MI and systolic heart failure with LV ejection fraction of 15% -Patient compliant with: carvedilol, furosemide, aspirin, lisinopril, and atorvastatin -Exam: elevated jugular venous pressure, bibasilar crackles, S3, bilateral lower extremity pitting edema -Started on dobutamine infusion= symptom relief -Mechanism of dobutamine? a)Cyclic nucleotide phosphodiesterase inhibition b)Gs protein-adenylate cyclase activation c)Late-phase inward sodium channel inhibition d)Sodium-potassium exchange pump inhibition e)Soluble guanylate cyclase activation f)Voltage-dependent calcium channel inhibition

answer: Gs protein-adenylate cyclase activation -Dobutamine (beta agonist; beta-1 mainly, weak beta-1 and alpha-1 agonist)= acts via Gs and cAMP -In cardiac myocytes; increased cAMP= calcium channel activation and increased cytosolic Ca2+ concentration= conformational change in troponin complex= facilitating actin-myosin binding and increasing myocardial contractility (positive inotropy) -Increased cAMP= also causes rise in heart rate, by increasing Na and Ca channel activation in pacemaker cells (positive chronotropy) -In vasculature= beta-2 > alpha-1= mild vasodilation -Increase in CO= improve symptoms and end-organ perfusion in patients with severe LV systolic dysfunction and cardiogenic shock option A= PDE inhibitors (e.g., milrinone)= increase in intracellular cAMP levels (prevent breakdown)= vasodilation and positive inotropy option C= Ranolazine is an anti-anginal drug that inhibit late phase inward sodium channels in ischemic myocardial cells during cardiac repolarization= decrease in intracellular sodium= enhances calcium efflux (Ca/Na exchanger)= reduced myocardial oxygen consumption option D= digoxin; inhibiting Na/K ATPase= increase intracellular sodium= decreases Na/Ca exchanger activity= increase intracellular Ca2+= improve myocardial contractility option E= nitrates; provide radical NO, which activates guanylate cyclase and increases cGMP levels= dephosphorylation of myosin light chains= SMC relaxation option F= CCBs= vasodilation and decrease in contractility

40 years woman -History: hypertension, depression -Found obtunded in her apartment, hypotensive, bradycardic -IV glucagon administered= conditions improve -Mechanisms a)Increased synaptic release of glutamate b)Decreased cAMP in vascular smooth muscle c)Decreased DAG in vascular smooth muscle d)Increased cAMP in cardiac myocytes e)Increased IP3 in cardiac myocytes

answer: Increased cAMP in cardiac myocytes -Overdosed on beta blocker medication (most likely) -Beta-blocker overdose= depression of myocardial contractility, bradycardia and varying degrees of AV block= low CO -Glucagon is the drug of choice for beta blocker overdose -Glucagon= Gs= increase cAMP= increase release of intracellular calcium during muscle contraction= increase HR and contractility (improvements in HR and BP within minutes)

63 years man -Chest pain and blockage of LAD artery (2 months ago)= Percutaneous coronary intervention with placement of a drug-eluting stent and discharged with medications -History: hypertension, T2D, and degenerative joint disease -No chest pain with medications, but has diffuse muscle aches and cramps, especially after exercise -Labs: elevated serum creatine kinase -Medication with such side effects causes? a)Decreased gastric mucosal prostaglandin synthesis b)Elevated plasma bradykinin level c)Impaired potassium entry into cells d)Increased hepatocyte LDL receptor recycling e)Increased renal calcium reabsorption f)Inhibition of hepatic gluconeogenesis

answer: Increased hepatocyte LDL receptor recycling -Most likely statin-induced myopathy -Statins= effective for hypercholesteremia and now routinely given with symptomatic CAD (regardless of baseline cholesterol levels) -Side effects= range from myalgia or myopathy with or without myonecrosis (elevated serum creatine kinase), to frank rhabdomyolysis -Statins= inhibit HMG CoA reductase= preventing conversion of HMG CoA to mevalonic acid (rate-limiting step in cholesterol biosynthesis)= decreased cholesterol= increased hepatic clearance of LDL from circulation by LDL receptors= after endocytosis of LDL= receptors recycle back to surface -The increase in LDL receptor recycling allows intrahepatic cholesterol levels to remain at normal levels while blood levels are kept low option A= NSAIDs option B= ACEIs option C= beta blockers= cause hyperkalemia option E= thiazide diuretics=hypercalcemia option F= metformin inhibits hepatic gluconeogenesis via inhibition of the mitochondrial isoform of glycerophosphate dehydrogenase in liver; side effects= GI upset and diarrhea

82 years man -Fatigue and palpitations (sudden) -History: asthma, gout and hypertension -High Pulse, rhythm is irregularly irregular -IV digoxin given; pulse is decreased but still irregularly irregular -Mechanism of lowering HR? a)Decreased action potential duration b)Decreased atrial refractoriness c)Delayed after-depolarizations d)Increased parasympathetic tone e)Increased ventricular contractility

answer: Increased parasympathetic tone -patient: atrial fibrillation with rapid ventricular response (RVR) and treated with digoxin (ventricular rate control); CCBs and beta blockers are preferred but digoxin can be used sometimes -Digoxin= slow ventricular rate by increasing parasympathetic tone= leads to inhibition of AV nodal conduction -Inhibition of Na/K ATPase pump in vagal afferent fibers sensitizes baroreceptors (e.g., carotid, aortic) and cardiac receptors, augmenting afferent input from the CVS to brain -Digoxin also enhances efferent parasympathetic ganglionic transmission, leading to increased vagal output -Slowed AV conduction is useful in AF with RVR (HR> 100); because focus of increased automaticity in these conditions is located in atria and rapidity of ventricular response depends on refractory period of AV node= when AV node is slowed, atrial will continue to fibrillate or flutter but ventricles will have normal rate -RVR= inadequate diastolic filling time= poor CO= heart failure option A= ectopic foci will fire at greater frequency; predispose to AF option B= give atria ability to depolarize at higher rate and increase risk of reentry; predispose to AF option C= are abnormal depolarizations of cardiac myocytes that occur after repolarization has completed; occur in states of hyperexcitability (e.g., very high intracellular calcium, high catecholamine states)= digoxin toxicity can result in DADs via its mechanism of increasing intracellular calcium= ventricular tachycardia= death option E= digoxin can improve systolic heart failure symptoms via this mechanism; inhibit Na/K ATPase= high Na= dysfunction of Na/Ca= high Ca= high contractility

56 years man -Heart failure with reduced ejection fraction -Hospitalized twice (last 3 months): acute decompensated heart failure that required treatment with IV furosemide -Since discharge= high dose oral furosemide, but continues to have pedal edema and weight gain -Addition of metolazone may assist patient with because of loop diuretics effect? a)Increased calcium delivery to the distal tubules b)Increased sodium delivery to the distal tubules c)Increased tubular reabsorption of bicarbonate d)Increased tubular reabsorption of uric acid e)Reduced tubular reabsorption of glucose f)Reduced tubular reabsorption of potassium

answer: Increased sodium delivery to the distal tubules -Patient has decompensated heart failure with ongoing volume retention despite high doses of oral furosemide -Thiazide diuretics, especially metolazone, potentiate the diuretic effect of loop diuretics (helps in refractory volume overload) -Sodium is the major determinant of volume status -Loop diuretics= block Na/K/2Cl in ascending limb= more sodium excreted (limited by reabsorption of sodium in DCT)= inhibition of Na/Cl in DCT by thiazides prevents reabsorption of increased sodium delivery option A= loop diuretics inhibit calcium absorption in loop of Henle option C= Loop diuretics stimulate hydrogen excretion and bicarbonate reabsorption= metabolic alkalosis option D= both loop and thiazide diuretics= increase risk of gout attacks option E= sodium-glucose cotransporter-2 inhibitors, e.g., empagliflozin= osmotic diuresis

76 years man -Severe midsternal chest pain and diaphoresis -History= hypertension, T2D, asymptomatic right carotid artery stenosis -ECG= ST elevation greater than 1mm in leads II, III and aVF -Patient receives aspirin immediately followed by alteplase and low dose-beta blocker -Single dose of morphine for pain control (IV) -Several hours later= comatose with asymmetric pupils and irregular breathing pattern -Cause? a)Carotid artery thrombosis b)Dissection of ascending aorta c)Interventricular septum perforation d)Intracerebral hemorrhage e)Myocardial reperfusion injury f)Opioid overdose g)Pulmonary embolism

answer: Intracerebral hemorrhage -Chest pain + diaphoresis + history of systemic atherosclerosis and T2D= acute coronary syndrome -ECG= inferior STEMI -PCI or fibrinolysis to achieve myocardial reperfusion is recommended for acute MI patients who present within 12 hours of onset of symptoms -PCI is preferred over fibrinolytic therapy due to lower rates of intracerebral hemorrhage and recurrent MI; it may not be available in all institutions -in such cases, fibrinolytics (alteplase, tenecteplase) improves clinical outcomes in absence of contraindications (e.g., GI bleeding, recent surgery) -Alteplase binds fibrin in the thrombus (clot) and converts entrapped plasminogen to plasmin= plasmin hydrolyzes key bonds in the fibrin matrix causing clot lysis and restoration of coronary arterial blood flow -most common adverse effect: hemorrhage, this patient has signs of intracerebral hemorrhage (decreased level of consciousness, asymmetric pupils and irregular breathing) option A= can cause ischemic stroke with fluctuating symptoms that may progress with periods of improvement; abrupt onset of symptoms along with loss of consciousness= ICH more likely option B= presents with severe, tearing chest pain that radiates to interscapular area; occurs most commonly in patients with hypertension, Marfan/EDS; X-ray= widened mediastinum option C= results in left-to right shunt and symptoms of acute heart failure; chest pain, dyspnea, symptoms of cardiogenic shock, harsh holosystolic murmur on left sternal border option E= results in paradoxical cardiomyocyte dysfunction; can occur in patients treated with PCI, thrombolytics or coronary artery bypass grafting; clinical manifestations= arrhythmias, myocardial stunning (prolonged but reversible contractile dysfunction) and myocyte death option F= miosis, respiratory depression= RR<12 option G= acute onset of dyspnea, pleuritic chest pain, and possibly hemoptysis; severe= tachycardia, hypotension, cyanosis, and loss of consciousness

66 years man -Recurrent syncope (3 episodes of dizziness, palpitations and loss of consciousness last 6 months) -History: hypertension and hyperlipidemia -Cardiac electrophysiological study: IV infusion medication= increased contractility, decreased vascular resistance -Medication? a)Acetylcholine b)Adenosine c)Clonidine d)Esmolol e)Isoproterenol f)Norepinephrine g)Phenylephrine

answer: Isoproterenol -Isoproterenol= nonselective beta agonist= increases cardiac contractility (beta-1) and vasodilation (decrease resistance; beta-2) option A= binds M2 in heart and M3 on vessels= decreases HR and conduction velocity and vasodilation (mediated by NO endothelial release); no effect on contractility option B= A1= activate potassium channels= hyperpolarization= delay in AV node; can also cause peripheral vasodilation, and slows down contractility option C= stimulates alpha-2 in brainstem= reduced sympathetic outflow from CNS= decrease in peripheral vascular resistance, HR and BP, no effect on contractility option D= blocks beta-1= negative inotropy and chronotropy option F= alpha- vasoconstriction; weak beta-1= modest increase in contractility option G= alpha adrenergic agonist= vasoconstriction only, no contractility effect

45 years man -Acute onset of severe chest pain and diaphoresis (squeezing pain) -History: hypertension -Father: coronary artery bypass grafting at 50 years -Diagnosis: acute ST elevation MI and undergoes urgent coronary intervention with stent placement into right coronary artery -Started on: high-intensity atorvastatin, antiplatelets, and medications for hypertension -4 weeks later= total cholesterol dropped to normal -Increased due to therapy? a)ApoB-100 concentration b)Biliary excretion of cholesterol c)LDL receptor density d)Plasma free fatty acids e)VLDL concentration

answer: LDL receptor density -Statins (HMG CoA inhibitors)= inhibit rate limiting step in cholesterol synthesis (HMG CoA to mevalonate)= increase surface expression of LDL receptor to increase uptake of circulating LDL (20-50% reduction in cholesterol and LDL) -High-intensity statin therapy reduces risk of acute coronary events -Statins also have anti-inflammatory properties, improve epithelial dysfunction, and stabilize atherosclerotic plaques option A= apoprotein present in VLDL and LDL; decreases in circulating LDL and VLDL= decrease ApoB-100 option B= decreases in patients on statins due to decreased de-novo cholesterol synthesis option D= statins lower triglycerides and minimal effect on plasma free fatty acid concentration option E= cause small decreases in VLDL; fibrates and nicotinic acid therapy decrease hepatic production of VLDL and are mainstay treatment for primary hypertriglyceridemia (increased VLDL)

48 years man -Diagnosed: hypercholesteremia (6 months ago)= follows dietary and lifestyle modifications -Father: diabetes, CAD -BMI is high; Labs: high LDL -What should be obtained before starting statin therapy? a)Apolipoprotein-B level b)Complete blood count c)Lipoprotein lipase activity assay d)Liver transaminase levels e)Serum cortisol level f)Serum creatinine

answer: Liver transaminase levels -Severe elevation in LDL + family history of CAD + familial hypercholesteremia= high risk of CVD -HMG-CoA reductase inhibitors (competitive; statins) are first-line therapy for most patients with hypercholesteremia= reduce risk of cardiac events= reduced cholesterol synthesis -Side effects: myopathy and hepatitis -Mild elevations of liver enzymes are common, significant liver injury (high transaminases) is seen in less than 1%= reversible if medication is discontinued -Liver function tests are recommended before starting statin therapy option A= found on atherogenic lipoprotein particles; measurements of Apo-B levels= quantify LDL particle count option C= lipoprotein lipase deficiency= can cause familial hypertriglyceridemia (heterozygous) and chylomicronemia (homozygous) syndromes= present with elevated triglycerides option E= cholesterol is a precursor in synthesis of adrenocortical and gonadal steroid hormones; statin medications in theory can cause hypogonadism or hypoadrenalism; clinically, however, there are no significant alterations in serum cortisol or gonadal steroid levels unless the patients also takes other adrenal enzyme inhibitors, like ketoconazole, aminoglutethimide option F= statins have no significant renal side effects; in rare instances, they can cause rhabdomyolysis, which can precipitate acute renal failure; occurs less commonly than hepatic dysfunction and would be associated with myalgias and myoglobinuria

53 years man -Shortness of breath and chest tightness (during poker tournament) -History: hypertension (noncompliant) -BP very high; bibasilar crackles -Nitroglycerine infusion started= BP dropped -Mechanism? a)Actin phosphorylation b)Calcium release from sarcoplasmic reticulum c)Enhanced cyclic mononucleotide degradation d)Inositol triphosphate accumulation e)Myosin dephosphorylation f)Tyrosine kinase activation

answer: Myosin dephosphorylation -presentation= acute pulmonary edema due to severe elevated BP (hypertensive emergency) -In such cases, IV vasodilators (nitroglycerine, sodium nitroprusside) are often used to improve the acute heart failure by reducing preload and afterload -Nitrates are metabolized within vascular SMCs to NO= activates guanylate cyclase= promotes conversion of GTP to cGMP= increased levels of cGMP= decreased intracellular calcium (reduce activity of myosin light chain kinase) and activation of myosin light chain phosphatase= promotes myosin light chain dephosphorylation and vascular smooth muscle relaxation option A= in smooth muscles, stimulus to contract leads to increased intracellular Ca2+, which leads to myosin phosphorylation; once phosphorylated it binds to actin and causes contraction option B= would lead to increased contraction of cardiac myocytes option C= enhanced contraction in smooth muscle cells option D= binds to receptor on SER= leads to release of Ca2+= smooth muscle contraction option F= insulin and insulin-like growth factor mechanism

47 years man -CAD, comes with chest tightness, sweating and palpitations (2 hours ago) -Six months ago= percutaneous angioplasty of RCA -History: anxiety and panic attacks -BP is high, pulse is high (regular) -Esmolol infusion produces rapid symptom relief and heart slowing -Where on ECG does it act on? a) T wave b) PR interval c) QRS complex d) ST segment e) QT interval

answer: PR interval -PR interval= atrial depolarization= the longer the PR interval, the longer it takes for an electrical impulse to travel from SA to ventricles via AV node -Drugs that slow AV nodal conduction= beta blockers, will prolong PR -Beta blockers (except sotalol, which is also class III) do not have specific effects on QRS or QT durations option A= myocardial ischemia can cause T wave inversion option C= class IC prolong QRS option D= ST elevation or depression= Myocardial ischemia option E= QT interval affected by class Ia and class III

54 years man -Paroxysmal atrial fibrillation -During episodes= palpitations, chest tightness, light headedness, shortness of breath -History: asthma, hypertension, diet-controlled diabetes mellitus -It is decided to proceed with Dofetilide therapy -Increased risk of? a)Bronchospasm b)Complete atrioventricular block c)Polymorphic ventricular tachycardia d)Thyroid dysfunction e)Venous thromboembolism f)Visual disturbance

answer: Polymorphic ventricular tachycardia -Young patient with symptomatic paroxysmal atrial fibrillation are often treated with a rhythm control strategy to restore and maintain normal sinus rhythm and to eliminate symptoms -Drugs commonly used for rhythm control of atrial fibrillation include Class IC (e.g., flecainide, propafenone) and Class III agents (e.g., Ibutilide, Dofetilide) -Dofetilide= block potassium channels= slow repolarization and prolong APD and effective refractory period= suppressing electrical foci that lead to atrial fibrillation -Side effect= QT prolongation= leading to risk of TdP (form of polymorphic VT)= can cause syncope or SCD= give drug in hospital only with cardiac monitoring option A= adenosine; due to release of histamine and leukotrienes option B= beta blockers and Nondihydropyridine CCBs; digoxin and adenosine can cause AV block too; sotalol and amiodarone (class III) can also slow AV conduction, but not Dofetilide and Ibutilide option D= amiodarone option F= digoxin; yellow tint or blurred vision

57 years man -Episode of acute pancreatitis (no gallstones) -History: severe hypertriglyceremia -Treated with fibrate medication (in past); can't tolerate anymore due to hepatic toxicity -Labs: normal hepatic and pancreatic enzyme levels, but elevated triglycerides -Prescribed medication that can cause skin flushing and warmth after taking pills -Primary agent mediating these effects? a)Histamine b)Platelet-activating factor c)Prostaglandin d)Serotonin e)Substance P

answer: Prostaglandin -Nicotinic acid, or niacin, (effective in raising HDL, lowering LDLs and triglycerides); used in treatment of hyperlipidemia -Side effects: cutaneous flushing, warmth, itching= mediated by release of prostaglandins (particularly PGD2 and PGE2) -Aspirin, which inhibits prostaglandin synthesis, can significantly reduce these side effects if given 30-60 mins before niacin administration -side effects are also reduced with slow-release preparations or if niacin is taken with meals -They are worst when niacin is first initiated and tend to fade over time due to tachyphylaxis option A= vancomycin-induced red man syndrome is mediated by histamine release due to non-IgE mediated mast cell degranulation option B= powerful phospholipid activator produced by neutrophils, basophils, platelets, and endothelial cells= mediator of platelet aggregation option D= serotonin syndrome is a potential side effect of SSRIs= abnormal mental status, autonomic hyperactivity, and muscular rigidity with hyperreflexia option E= involved in mediating pain signals in the PNS and CNS; topical capsaicin causes release of substance P, resulting in burning pain in area of application; prolonged use causes depletion of substance P and the pain diminishes with time

42 years man -Episode of syncope; intermittent palpitation (last few weeks), sometimes with lightheadedness -History: chronic lower back pain that began after falling from ladder (3 years ago; fractured lumbar vertebrae) -Back pain= uncontrolled with conservative measures, treated with escalating doses of methadone for past year -ECG findings? a)Atrioventricular block b)Deep Q waves c)QT interval prolongation d)Sinus pauses e)Ventricular preexcitation

answer: QT interval prolongation -Most likely has QT prolongation, common adverse effect of methadone (inhibits delayed rectifier potassium current) -QT interval prolongation= due to delayed ventricular repolarization, which occurs due to impairment of voltage-gated potassium channels that control delayed rectified potassium current= can trigger torsades de pointes, a form of polymorphic ventricular tachycardia named for a repetitive twisting of the peaks pattern= serious arrhythmia that can cause palpitations, lightheadedness, syncope and sudden cardiac death option A= advanced AV block, i.e., Mobitz II or complete AV block= can cause syncope; causes: infections (e.g., Lyme disease), ischemia, and medications (e.g., beta blockers, Nondihydropyridine CCBs) option B= represent previous MI; patients with history of MI= risk of monomorphic ventricular tachycardia causing syncope or SCD, but unlikely in this patient option D= sick sinus syndrome, results from age-related degeneration of the conduction system; usually >65 years and may experience progressive fatigue and episodes of syncope option E= are recognized by upslope of the QRS complex (delta wave) on ECG; patients with this Wolf-Parkinson White syndrome may develop paroxysmal supraventricular tachycardia; can present with syncope or SCD; onset usually <40 years

36 years man -Sedentary lifestyle, eats mostly fast food, rarely exercise, and drinks daily, high BMI -Labs: high total cholesterol, and triglycerides -He is started on fenofibrate therapy -Mechanism? a)Blocking intestinal cholesterol absorption b)Decreasing hepatic cholesterol synthesis c)Increasing fecal loss of cholesterol derivatives d)Inhibiting LDL receptor degradation e)Reducing hepatic VLDL production

answer: Reducing hepatic VLDL production -Elevated triglyceride level= increased risk of CVD -Lipoprotein lipase hydrolyzes triglycerides in chylomicrons and VLDL to release free fatty acids, which can be used for energy or converted back to triglycerides for storage in adipose tissue -it also facilitates transfer of triglycerides from these lipoproteins to HDL -Fibrates (e.g., gemfibrozil, fenofibrate) activate peroxisome proliferator-activated receptor alpha (PPAR-alpha) which leads to decreased hepatic VLDL production and increased LPL activity -Decreased triglycerides (25-50%) and increase HDL (5-20%) -Omega-3 fatty acids also decrease VLDL production and inhibit synthesis of apolipoprotein B as well= lower triglycerides -Niacin also decreases VLDL production (increases HDL and lowers triglycerides) option A= Ezetimibe option B= statins option C= bile acid resins like cholestyramine option D= proprotein convertase subtilisin kexin 9 (PCSK9) inhibitors are monoclonal antibodies that reduce LDL receptor degradation

62 years man -Hospitalized for chest pain and palpitations -History: CAD, ischemic cardiomyopathy, paroxysmal atrial fibrillation and hypertension -During hospitalization= 2 episodes of brief loss of consciousness -ECG= polymorphic QRS complexes that change in amplitude and cycle length= QT prolongation -Medication? a)Digoxin b)Diltiazem c)Lidocaine d)Metoprolol e)Sotalol

answer: Sotalol -ECG findings= long QT syndrome with associated torsades de pointes (TdP) -TdP= form of polymorphic ventricular tachycardia characterized by QRS complexes of varying amplitude and cycle length (different from other PVT because it is always associated with long QT interval); it may terminate spontaneously or can degenerate into ventricular fibrillation and SCD -Acquired or congenital -Acquired QT prolongation= 1) electrolyte imbalances (hypokalemia, hypomagnesemia) 2) medications (class IA, class III antiarrhythmics; e.g., quinidine, sotalol) 3) antibiotics (e.g., macrolides, fluoroquinolones) 4) methadone 5) antipsychotics (e.g., haloperidol) -Sotalol= class III= for atrial fibrillation= blocks K channels= prolongs AP (so prolongs QT) option A= slows conduction through AV node by augmenting vagal parasympathetic tone; it increases cardiac contractility by inhibiting Na/K ATPase; decreases APD and can cause QT shortening option B and D= slow sinus node discharge rate and AV nodal conduction; prolong PR interval option C= Class IB; blocks sodium channels and inhibit phase 0 depolarization

65 years man -Substernal chest discomfort and intermittent palpitations -History: CAD and stable angina (managed medically) -Medications: high dose metoprolol, atorvastatin, low-dose aspirin -Traveling and ran out of medication -High BP and pulse; ECG= tachycardia with a 1mm ST segment depression in lateral leads -Cause of presentation? a)Increased levels of circulating catecholamines b)Predominant activation of alpha-adrenergic receptors c)Reduced activity of cyclic AMP-mediated massaging d)Reduced cellular sensitivity to circulating catecholamines e)Upregulation of beta-adrenergic receptors

answer: Upregulation of beta-adrenergic receptors -Elevated BP, tachycardia, palpitations, and anginal pain following discontinuation of metoprolol= beta blocker withdrawal syndrome -With changes in environmental stimulus, feedback mechanisms in the cells adjust the density of cell surface receptors to regulate sensitivity to stimulus -Prolonged beta blockade, for example, stimulates increase in receptor expression (upregulation)= when suddenly stopping beta blockers= increased receptors= amplified response -Metoprolol is Cardioselective and primarily blocks beta-1 receptors, abrupt cessation= increase HR and contractility= increased BP due to increased CO -These changes= increase oxygen demand that may cause ischemia (evidenced by ST depression on ECG= trigger angina in patients with underlying CAD option C= this response is increased in withdrawal

78 years man -Severe dementia; lethargy, fever, and vomiting (one day) -History: emphysema, heart failure with reduced ejection fraction, hypertension and T2D -Exam: low BP, high pulse; extremities are warm; coarse rhonchi in right lower lung -Given IV infusion of an agent that increases peripheral vascular resistance, increases BP and decreases HR -Agent given? a)Dobutamine b)High-dose dopamine c)Low-dose epinephrine d)Milrinone e)Phenylephrine

answer: phenylephrine -Adrenergic agonists (pressors) are often used to increase BP in patients with shock (especially septic shock) -Phenylephrine is a selective alpha-1 agonist with no effect on alpha-2 receptors or beta-receptors= vasoconstriction (increased SVR and BP), but no direct effect on heart -Indirect effect on heart= baroreceptor reflex= increased BP stimulates firing of aortic and carotid baroreceptors= increased PSNS response and decreased SNS output= decreases HR and myocardial contractility option A= mostly beta-1 agonist; beta-1= increase HR and contractility; beta-2= vasodilation; useful in cardiogenic shock option B= alpha-1> beta-1> D1; alpha-1= increases SVR and BP; beta-1 stimulation offsets the reflex mediated decrease in HR= HR relatively unchanged option C= beta-1> beta-2> alpha-1; beta-1= increase contractility and HR; beta-2= vasodilation more than vasoconstriction of alpha-1 option D= inhibits PDE-3 to increase myocardial contractility and causes peripheral vasodilation (decreases SVR); useful in cardiogenic shock

24 years man -Sudden onset palpitations (similar episode year ago that resolved spontaneously) -Very high pulse; rapid IV medication instantaneously resolves arrhythmia -Accompanied by short-lived flushing, burning in chest and shortness of breath -Medication used? a)Adenosine b)Amiodarone c)Digoxin d)Ibutilide e)Lidocaine f)Verapamil

answer: adenosine -Symptoms consistent with paroxysmal supraventricular tachycardia (PSVT)= comes suddenly and the focus of automaticity lies above the ventricles (e.g., atria, AV node)= due to development of a reentrant circuit, the tachyarrhythmia may be terminated with adenosine -Adenosine is a coronary arteriole vasodilator, but it also affects cardiac conduction -Antiarrhythmic= acts by hyperpolarizing the nodal pacemaker and conducting cells to briefly block conduction through AV node= blocks conduction from atria to ventricles= terminate reentrant circuit PSVT -Effects are rapid-onset and short-lived (half-life <10secs) -Adverse effects: flushing, chest burning (due to bronchospasm), hypotension, and high-grade AV block option B= Class III; can be used for supraventricular (e.g., PSVT, Afib) and ventricular arrhythmias; adverse effects: photodermatitis, blue/gray skin discoloration, pulmonary fibrosis, liver toxicity, hyper or hypothyroidism option C= slows conduction through AV= treat SVT, especially atrial fibrillation; toxicity= fatigue, yellow-tinted vision, nausea/vomiting, diarrhea, confusion option D= class III; in atrial fibrillation and flutter; adverse effects: QT prolongation option E= class IB; used for ventricular arrhythmias post MI; toxicity= neurological symptoms or arrhythmias option F= class IV; sometimes used in SVT; adverse effects= bradycardia, constipation, and AV block and can exacerbate heart failure

45 years man -Severe chest pain, sweating and nausea (hour ago) -Father: died at age 50, after sudden onset chest pain -Smoker (daily); ECG: ST elevation in leads II, III and aVF -After initial treatment= chest pain decreases, and reperfusion complex ventricular arrhythmia emerges -Arrhythmia is asymptomatic and resolves spontaneously? -Drug causing rapid reperfusion in patient? a)Alteplase b)Apixaban c)Argatroban d)Aspirin e)Heparin f)Prasugrel g)Rosuvastatin

answer: alteplase -Patient had STEMI and, following medical treatment= symptomatic improvement and reperfusion-related arrhythmia, this is most likely result of fibrinolytic (thrombolytic) therapy -Fibrinolytic agents (e.g., alteplase) are indicated in patients with acute STEMI who can't receive percutaneous coronary intervention on time -Administration= breakdown of fibrin clot and often restoration of myocardial perfusion, some patients develop a self-limiting reperfusion-related arrhythmia (most commonly accelerated idioventricular rhythm) -In the fibrinolytic pathway, tissue plasminogen activator (tPA) is released by the endothelium and cleaves plasminogen to form plasmin= plasmin then acts to both degrade individual fibrinogen molecules and break down fibrin clot that has already formed= leads to elevated fibrin split products (e.g., d-dimer) in blood -Pharmacological fibrinolytic agents (streptokinase, and recombinant forms of tPA (e.g., alteplase, tenecteplase)= increased bleeding risk and contraindicated in patients with increased risk of bleeding complications (e.g., recent hemorrhage) option B= direct factor Xa inhibitor; treatment and prevention of venous thromboembolism; oral anticoagulant option C= direct thrombin inhibitor used to treated heparin-induced thrombocytopenia; IV anticoagulant option D= inhibits cyclooxygenase to reduce thromboxane levels; irreversible inhibition of platelet aggregation; used with P2Y12 receptor blocker to treat acute STEMI to prevent additional platelet thrombus; however, they don't cause lysis of existing thrombus option E= IV anticoagulant; potentiates activity of antithrombin III to inhibit thrombin and factor Xa and exert anticoagulant effect; used in acute STEMI to prevent further clot formation; doesn't cause thrombolysis option F= P2Y12 receptor blocker (e.g., clopidogrel, prasugrel) prevent binding of adenosine diphosphate to platelets; irreversible inhibition of platelet aggregation; used with aspirin to treat acute STEMI to prevent additional platelet thrombus; however, they don't cause lysis of existing thrombus option G= statin; used in acute STEMI= stabilize atherosclerotic plaque, reduce vascular inflammation, promote long-term regression of atherosclerosis; doesn't cause thrombolysis

60 years man -Dizziness and palpitations -While mowing lawn= lightheaded and had to sit to avoid passing out -After antiarrhythmic= prolong QT interval, but associated with lower incidence of torsades de pointes -Medication? a)Adenosine b)Amiodarone c)Digoxin d)Esmolol e)Lidocaine f)Procainamide g)Verapamil

answer: amiodarone -Amiodarone= class III (mimics class I, II, III and IV)= for management of SVT and ventricular arrhythmias -Blocks potassium channels= prolong APD and QT interval (class Ia also do that by blocking Na channels and prolonging APD and QT)= associated with TdP -Amiodarone= very little risk of TdP (Ranolazine also prolongs QT and associated with no risk of TdP)

77 years man -High BP (put on medication to treat hypertension) -Follow-up visit (3 weeks later)= BP is slightly elevated (but dropped) and 2+ pitting lower extremity edema -Medication used to treat hypertension? a)Amlodipine b)Eplerenone c)Hydrochlorothiazide d)Ramipril e)Torsemide f)Valsartan

answer: amlodipine -Dihydropyridine calcium channel blockers (e.g., amlodipine, nifedipine)= effective in hypertension (monotherapy or combination therapy) -Side effects: headache, flushing, dizziness, and peripheral edema -Mechanism of edema= dilation of precapillary vessels (arteriolar dilation)= increased capillary hydrostatic pressure and fluid extravasation into the interstitium -ACEIs or ARBs cause postcapillary venodilation= can normalize capillary hydrostatic pressure= reducing risk of peripheral edema in patients taking CCBs option B= aldosterone antagonist; hyperkalemia, increased creatinine and gynecomastia/1% compared to spironolactone option C and E= hypokalemia, hyponatremia, hypomagnesemia option D= cough, hypotension, hyperkalemia, angioedema option F= same as D, but risk of angioedema and cough is lower

65 years man -Follow up of CHF -Missed appointment due to upset stomach; history: disturbed color perception, anorexia, nausea, vomiting, and diarrhea (2 weeks) -If symptoms are not controlled, which of the following complications might develop? a)Angioedema b)Arrhythmia c)Bronchoconstriction d)Hypotension e)Pulmonary edema

answer: arrhythmia -Most likely= digoxin toxicity -Most serious complication= cardiac arrhythmias of any type -Narrow therapeutic index= toxicity common -Precipitating causes of arrhythmias: hypokalemia (increased digoxin binding), hypovolemia, and renal failure -Treatment: anti-digoxin antibody fragments option A= ACEIs option C= nonspecific beta blockers in patients with COPD or asthma option D= can occur with digoxin toxicity due to bradyarrhythmias and severe vomiting, diarrhea= hypovolemia; but digoxin is inotropic positive= increases CO; so hypotension is a variable manifestation of digoxin toxicity option E= uncontrolled heart failure

60 years man -Hypertension -Treated with lisinopril but stopped a weak ago (dry cough) -History: T2D with moderately increased albuminuria but normal creatinine clearance -Other medications: metformin, rosuvastatin -High BP and BMI -Best drug for treatment of patient hypertension? a)Diltiazem b)Hydralazine c)Metoprolol d)Ramipril e)Valsartan

answer: valsartan -Diabetes causes impaired autoregulation of glomerular blood flow= elevations of intraglomerular pressures and chronic glomerular injury -Diabetes + hypertension (at risk for chronic kidney disease) -ACEIs (e.g., lisinopril) cause preferential dilation of efferent arterioles, lowering intraglomerular pressures and reducing risk of chronic glomerular injury -ACEIs= inhibit converting AT I to AT II, but also degrades bradykinin and substance P= elevated bradykinin and substance P= nonproductive cough -ARBs have similar hemodynamic effects but do not act on bradykinin or substance P= no cough option D= replacement with another ACEI= can cause same cough

55 years man -Hour of intense substernal chest burning accompanied by sweating -History: obesity and diet controlled T2D -ECG: 2mm ST elevation in leads II, III and aVF -Aspirin and morphine are administered -As patient is about to be transported to catheterization lab for urgent PCI, he develops dizziness -Telemetry= sinus bradycardia -Pale, diaphoretic and peripheral pulses are faint, low BP -Best immediate step for managing condition? a)Adenosine b)Alteplase c)Amiodarone d)Atropine e)Naloxone f)Physostigmine

answer: atropine -Acute inferior STEMI= due to occlusion of RCA (supplies SA, AV nodes, most of bundle of His)= conduction impairment (e.g., sinus bradycardia, AV block)= most common in inferior MI -Sinus bradycardia (50% of inferior MI patients)= due to nodal ischemia and enhanced vagal tone triggered by infarction of myocardial tissue (usually transient and resolves with restoration of coronary blood flow); however, patients with hypotension or symptoms of bradycardia (e.g., lightheadedness, syncope) should be managed with IV atropine (anticholinergic; counteracts enhanced vagal tone) option A= hyperpolarizes AV node; useful in paroxysmal SVT; worsens bradycardia and AV block option B= fibrinolytic agent; used in patients with STEMI that can't undergo PCI in a timely manner; PCI is preferred= less risk of intracranial hemorrhage option C= class III; management of ventricular arrhythmias in patients with acute MI; causes negative chronotropy option E= opioid receptor antagonist= reverse effects of opioid overdose; although bradycardia can be due to morphine, here it is more likely due to acute MI option F= acetylcholinesterase inhibitor= prolongs effects of Ach= used in myasthenia gravis= would worsen bradycardia

85 years man -fever, confusion -Admitted to ICU for presumptive diagnosis of septic shock -Antibiotics started -History: cardiovascular disease, diverticulitis, dementia -Low BP despite aggressive IV hydration; IV norepinephrine is administered in response to patient's hypotension -Cellular changes due to direct response to the medication? a)cAMP decrease in bronchial smooth muscle cells b)cAMP increase in cardiac muscle cells c)cAMP increase in vascular smooth muscle cells d)DAG decrease in vascular smooth muscle cells e)IP3 increase in cardiac muscle cells

answer: cAMP increase in cardiac muscle cells NE, adrenergic agonist= preferred treatment for septic shock as it predominantly stimulates alpha-1, alpha-2 and beta-1, with little influence on beta-2 -Stimulation of alpha1= vasoconstriction in skin and viscera (increased BP); via IP3 -Stimulation of beta-1= increase cAMP= increased contractility, conduction and HR (HR counteracted by reflex bradycardia due to alpha-1 stimulation= unchanged or decreased HR) Stimulation of alpha-2=decreaseincAMPinpancreaticbeta cells and in intestines= decreased insulin secretion and reduced intestinal motility; central alpha-2 receptors are not stimulated by IV NE (doesn't cross BBB) option A= beta-2 stimulation= increase cAMP option C= via IP3 not cAMP option D= increases IP3 and DAG not decreases it option E= occurs in vessels not heart

56 years woman -Chronic cough (dry) -History: long standing hypertension, diabetes and MI (2 months ago) -BP (130/70); best next step in management of this patient? a)Careful review of family history b)Careful review of past allergic episodes c)Careful review of current medications d)Careful review of diet and physical activity e)Chest X-ray

answer: careful review of current medications -History of hypertension, diabetes and CAD= likelihood that she takes ACEIs is high -Side effect of ACEIs= dry cough (high level of bradykinin, substance P or prostaglandin) -ACEIs block degradation of bradykinin and substance P= accumulate= inflammation and stimulate lung irritation -Switch ACEIs to ARBs= less likely to produce dry cough

68 years man -Progressive exertional dyspnea and lower extremity edema (several weeks) -History: non-Hodgkin lymphoma (remission) -Bibasilar lung crackles and 1+ bilateral lower extremity edema -Echo= biventricular dilation and LV ejection fraction of 35% -Stress myocardial perfusion scan= negative for inducible ischemia -After initial stabilization, long-term use of which of the following medications will most likely improve survival in this patient? a)Amiodarone b)Amlodipine c)Carvedilol d)Digoxin e)Diltiazem f)Flecainide g)Furosemide

answer: carvedilol -Clinical presentation: decompensated systolic heart failure due to nonischemic cardiomyopathy, likely as a result of chemotherapy for NHL -After initial stabilization, long term use of beta blockers (e.g., carvedilol, metoprolol) has been shown to improve survival in patients with HF due to LV systolic dysfunction -Beta blockade= decreases myocardial work and oxygen demand by slowing the ventricular rate and reducing contractility; it also lower peripheral resistance (afterload) by decreasing circulating levels of vasoconstricting hormones (e.g., renin, endothelin) -These effects are cardioprotective and help reduce cardiomyocyte death and limit deleterious cardiac remodeling -Beta blockers should not be initiated with patients with unstable (decompensated) HF, as they can further impair CO, they should be introduced after the patient has been stabilized -Other drugs that improve long-term survival in patients with systolic HF include: ACEIs, ARBs, and aldosterone antagonists option A= class III= for supraventricular and ventricular arrhythmias option B and E= CCB; does not improve survival in HF option D= used as adjunctive; improves symptoms and reduces rate of hospitalization; doesn't improve overall survival option F= class IC= occasionally for supraventricular arrhythmia management option G= first-line agents for rapid symptom control in patients with acute congestive HF; do not improve long-term survival

47 years man -History: hypertension, stable angina (takes sublingual nitroglycerine) -high BP; Patient started on low dose atenolol -Effects of atenolol? a)Cardiomyocyte cAMP: decreased; Juxtaglomerular cell cAMP: decreased; Vascular smooth muscle cAMP: decreased b)Cardiomyocyte cAMP: decreased; Juxtaglomerular cell cAMP: decreased; Vascular smooth muscle cAMP: no significant change c)Cardiomyocyte cAMP: decreased; Juxtaglomerular cell cAMP: no significant change; Vascular smooth muscle cAMP: decreased d)Cardiomyocyte cAMP: decreased; Juxtaglomerular cell cAMP: no significant change; Vascular smooth muscle cAMP: no significant change e)Cardiomyocyte cAMP: no significant change; Juxtaglomerular cell cAMP: no significant change; Vascular smooth muscle cAMP: no significant change

answer: choice B (Cardiomyocyte cAMP: decreased; Juxtaglomerular cell cAMP: decreased; Vascular smooth muscle cAMP: no significant change) -Beta blockers are the preferred treatment for patients with chronic stable angina and hypertension -Beta blockers= inhibit SNS= reduce HR and contractility (less myocardial oxygen demand) -At low doses, atenolol is a selective beta-1 adrenergic agonist -Beta-1 in cardiac and renal tissue (juxtaglomerular cells)= beta blockade= decreased cAMP levels in cardiac and renal tissue -In renal tissue it inhibits renin release

68 years man -Thigh and leg pain (worsens with exertion) -Unable to walk through the local mall with his wife without discomfort -History: hypertension and diabetes, smoker (daily) -Exam: weak dorsalis pedis pulses in both feet -Further evaluation= moderate peripheral artery disease involving both lower extremities -Drugs that would best provide symptomatic improvement due to direct dilation of arteries and inhibition of platelet aggregation? a)Abciximab b)Argatroban c)Aspirin d)Cilostazol e)Heparin f)Tissue plasminogen activator g)Warfarin

answer: cilostazol -thrombin, ADP, and thromboxane A2= activate platelets by acting on cell surface receptors -cAMP activates protein kinase A= inhibition of platelet aggregation -Agents that increase intra-platelet cAMP levels decrease platelet aggregation by preventing platelet shape change and granule release -Cilostazol reduces platelet activation by inhibiting platelet phosphodiesterase (enzyme responsible for breakdown of cAMP)= also a direct arterial vasodilator -Net effect= decrease in claudication symptoms and an increase in pain-free walking distances in patients with peripheral arterial disease option A= monoclonal antibody that inhibits platelet aggregation by targeting platelet IIb/IIIa receptor option B= direct thrombin inhibitor primarily used in treatment of heparin-induced thrombocytopenia irreversible cyclooxygenase 1 and 2 inhibitor= reduce risk of cardiovascular events; patients with PAD should take antiplatelet agents (aspirin, clopidogrel) to reduce risk of cardiovascular events= secondary prevention; but do not improve PAD symptoms option E= anticoagulant that potentiates antithrombin III= inactivation of thrombin option F= alteplase; converts plasminogen to plasmin and causes fibrinolysis option G= oral anticoagulant; competitive inhibitor of vitamin K epoxide reductase and reduces production of vitamin K dependent coagulation factors II, VII, IX, X, protein C and protein S= used in long-term treatment and prevention of thromboembolic events, doesn't reduce claudication symptoms

50 years man -Dizziness and confusion -Episode of chest pain and took nitroglycerine -Medications: daily aspirin for heart attack prevention, occasional acetaminophen for headaches and tadalafil for erectile dysfunction -BP (50/20) and HR (120) -Cellular changes responsible for patients' symptoms? a)Receptor downregulation b)Gs protein phosphorylation c)Cyclic GMP accumulation d)Tyrosine kinase overactivity e)Enhanced phospholipid metabolism f)Tolerance development

answer: cyclic GMP accumulation -Interaction between nitrates (nitroglycerine) and phosphodiesterase inhibitors (tadalafil, sildenafil, vardenafil) -Nitrates= converted to NO by vascular smooth muscle cells, and NO causes increased intracellular cGMP as second messenger= SMC relaxation -Additionally, cGMP is metabolized within cells by phosphodiesterase, and PDE inhibitors= increased cGMP levels -= cGMP accumulation= profound hypotension option A= or tachyphylaxis occurs with many drug classes including topical glucocorticoids, opiates, L-DOPA and more; one mechanism of developing tolerance option B= cAMP levels increase option D= would lead to exaggeration of effects of insulin or inulin-like growth factor option E= would lead to increase in production of prostaglandins (via diacylglycerol) and IP3 (leading to Ca2+ release from SER); mediates effects of alpha-1, angiotensin II, oxytocin and vasopressin receptors option F= refers to needing larger dose for same effect; multiple mechanisms: receptor downregulation, decreased response to receptor stimulation, increased rate of drug elimination by body

76 years woman -Lethargy, confusion (2 days) -Persistent atrial fibrillation and congestive heart failure (on multiple medications) -Nausea, decreased appetite, 3 episodes of vomiting -Vision difficulties, hyperkalemia -ECG: junctional escape rhythm at a rate of 48/min with occasional ventricular premature beats -Increased blood levels of which medication? a)Amiodarone b)Aspirin c)Digoxin d)Diltiazem e)Furosemide f)Metoprolol g)Spironolactone h)Valsartan

answer: digoxin Atrial fibrillation + CHF with signs of digoxin toxicity (CCBs and beta blockers are preferred treatments for patients with Afib with rapid ventricular response, digoxin is a common second-line treatment, particularly useful in patients with underlying systolic cardiac dysfunction); effects of digoxin -Increased vagal tone= slowing of conduction through AV node (rate-control) -Na/K ATPase inhibition= accumulation of sodium and calcium intracellularly (increased cardiac contractility) Digoxin has a narrow therapeutic index (toxicity common); symptoms= nausea, abdominal pain, fatigue, dizziness, confusion, blurred vision, yellow tint vision Toxicity can also cause cardiac arrhythmias= bradycardia, junctional escape beats due to increased AV nodal block Hypokalemia can precipitate toxicity by increasing digoxin binding to Na/K ATPase; however, elevated potassium is a sign of digoxin toxicity (due to inhibition of Na/K ATPases= potassium accumulates extracellularly) option A= pulmonary fibrosis, thyroid dysfunction, cardiac arrhythmias, elevated liver enzymes, visual disturbances, doesn't cause hyperkalemia option B= overdose= vertigo, tinnitus, vomiting, diarrhea; toxicity= coma, hyperpyrexia, pulmonary edema, and death option D and F= bradycardia, hypotension, AV block option E= volume depletion, hypokalemia, hypomagnesemia option G= hyperkalemia, gynecomastia, impotence, decreased libido option H= hypotension, renal failure, hyperkalemia

34 years man -Non-Hodgkin lymphoma undergoes doxorubicin containing chemotherapy -Weeks later= progressive exertional dyspnea; difficulty sleeping flat at night and needs multiple pillows to fall asleep -Cause of patient's symptoms? a)Dilated cardiomyopathy b)Focal myocardial scarring c)Hypertrophic cardiomyopathy d)Pericardial thickening and fibrosis e)Restrictive cardiomyopathy f)Right ventricular pressure overload

answer: dilated cardiomyopathy -Anthracyclines (e.g., daunorubicin, doxorubicin) are chemotherapeutic agents associated with severe cardiotoxicity than manifests as dilated cardiomyopathy -The toxicity is dependent on cumulative dose of anthracycline, affect patients develop symptoms of heart failure (e.g., dyspnea on exertion, orthopnea, peripheral edema) anywhere from several weeks to years following treatment -Mechanism: anthracyclines bind topoisomerase II enzyme in cancer cells to create DNA cleavage complexes that break DNA and cause cell death -Cardiotoxicity: 1) similar anthracycline-topoisomerase II complex that forms in healthy cardiomyocytes 2) anthracycline can form iron-containing complexes that produce DNA-damaging free radicals -Prevention of cardiomyopathy: administration of dexrazoxane, a chelating agent that blocks iron and topoisomerase II complexes in healthy cardiomyocytes option B= from MI option C= autosomal dominant; mutation of beta-myosin heavy chain option D= following cardiac surgery, radiation therapy, or viral infections of pericardium option E= initial cardiac manifestation of infiltrative diseases (e.g., hemochromatosis, amyloidosis, sarcoidosis) or radiation therapy option F= with acute pulmonary embolism; malignancy= risk for venous thromboembolic disease, but orthopnea doesn't explain pulmonary embolism

82 years man -Syncopal episode; recent onset constipation -Hospitalized (2 weeks ago): atrial fibrillation with rapid ventricular response= discharged with medication -History: hypertension, severe COPD -ECG= new onset second degree AV block; drug used for his condition? a)Amlodipine b)Diltiazem c)Hydrochlorothiazide d)Lidocaine e)Propranolol f)Terazosin g)Valsartan

answer: diltiazem -Constipation + new AV block (causing syncope)= adverse effects of Nondihydropyridine CCBs (e.g., diltiazem, verapamil) -These drugs uses= hypertension, angina, supraventricular arrhythmias (including atrial fibrillation) -Mechanism: block L-type calcium channels= decreasing phase 0 depolarization and conduction velocity in SA and AV nodes= slowing sinus rate and slowing conduction via AV node= bradycardia and varying degrees of AV block -Also have negative inotropic effect (relatively contraindicated in patients with LV systolic dysfunction) -Constipation (more common in verapamil)= due to reduced contraction of colonic smooth muscle option A= dihydropyridine CCB; predominant vasodilator effect= adverse effects= lightheadedness, flushing and peripheral edema option C= thiazide diuretic= for hypertension= adverse effects= hypokalemia and hyponatremia option D= Class IB= for symptomatic ventricular arrhythmias post MI option E= beta blocker= heart rate control in atrial fibrillation= can cause AV block; but also impairs bronchoconstriction= avoided in COPD= constipation also not adverse effect option F= alpha-1 antagonist; used in benign prostatic hyperplasia and hypertension; adverse effects= lightheadedness and orthostatic hypotension option G= ARB; used in hypertension and heart failure with reduced ejection fraction= adverse effects= hypotension and hyperkalemia

44 years man -Exertional shortness of breath and palpitations -Exam: systolic murmur at left sternal border and cardiac apex (gets louder when he stands up -Diagnosis= obstructive hypertrophic cardiomyopathy and paroxysmal atrial fibrillation -Medication changes in ventricular AP shown -Medication? (learn don't answer) a)Adenosine b)Digoxin c)Diltiazem d)Disopyramide e)Flecainide f)Lidocaine g)Propranolol

answer: disopyramide -figure= predominant slowing of phase 0 depolarization (block Na+ channels) and prolongation of phase 3 repolarization (moderately block K+ channels) after drug= prolonged duration of AP= class Ia antiarrhythmics (quinidine, procainamide, disopyramide) option A= activates potassium channels and increases potassium conductance by interacting with A1 receptors on cardiomyocytes= membrane hyperpolarization= slowing of SA rate and increase in AV conduction delay option B= inhibits Na/K ATPase= Na accumulation intracellularly= inhibition of Na/Ca2+= Ca2+ accumulation= better contractility ; also, it increases vagal output to AV node and conduction system= slowing conduction in these tissues option C= class IV= inhibit L-type calcium channels= slowing depolarization in SA and AV nodes and decreasing calcium entry into cardiomyocytes= slow sinus rate, prolong AV node conduction= depress myocardial contractility option E= Class IC; strongly block Na channels; use dependence (more effective at higher rates of depolarization); no affect on AP duration option F= Class IB; weak sodium channel blocking activity at rest; effective in blocking sodium channels in depolarized cardiac myocytes (used in ischemic tissue= after MI); shorten phase 3 repolarization option G= beta blocker, type II; slow conduction of AV node and prolongs phase 4 depolarization in pacemakers

65 years woman -History: T2D, hypertension -Occasional numbness in feet -Medication: ibuprofen for chronic back pain, along with hydrochlorothiazide, metformin, atorvastatin and insulin detemir -Urinalysis: albuminuria, normal creatinine -Lisinopril is initiated= patient returns due to lightheadedness and near-syncope (BP much lower on standing than in supine) -Major factor contributing to symptom? a)Autonomic neuropathy b)Chronic nonsteroidal anti-inflammatory drug use c)Diabetic nephropathy d)Diuretic therapy e)Unilateral renal artery stenosis

answer: diuretic therapy -Patient with albuminuria was started on ACEI for treatment of early diabetic nephropathy -Most patients remain asymptomatic with mild reduction in BP, but first-dose hypotension, can be a limiting factor when initiating ACEIs -Significant hypotension more likely in patients with high renin activity (such as those with volume depletion= from diuretic use= hydrochlorothiazide in this patient) or heart failure -Initiation of ACEIs= abrupt removal of vasoconstrictive effects of AT II= decreased peripheral vascular tone and drop in BP -To prevent development of first-dose hypotension= start at low dose then titrate upwards option A= numbness of feet= suggestive of diabetic peripheral neuropathy; at risk of developing autonomic neuropathy= can present with orthostatic hypotension; but here hypotension started on lisinopril therapy option B and C= albuminuria= early diabetic nephropathy= can be exacerbated by NSAIDs (reduce prostaglandin synthesis and increase resistance in afferent arterioles); ACEIs cause further decrease in GFR (efferent arteriolar vasoconstriction) and can precipitate acute renal failure in patients with underlying renal insufficiency; however, this patient has normal creatinine level= relatively intact renal function option E= ACEIs can worsen renal impairment in patients with bilateral renal artery stenosis, but usually not in those with unilateral renal artery stenosis

-Experiment: drug infused IV over different dose ranges and hemodynamic parameters were measures -Graphs are shown -Drug most likely used in this experiment? (learn don't answer) a) dopamine b) edrophonium c) epinephrine d) esmolol e) phenylephrine

answer: dopamine -low doses: stimulates D1 receptors in renal vasculature and tubules= increase in renal blood flow; GFR, and sodium excretion (can also vasodilate mesenteric, cerebral, coronary vascular beds) -Intermediate doses: stimulates beta-1 receptors in heart= increasing CO, heart rate, contractility and SBP -High dose: stimulation of alpha-1 receptors in systemic vasculatures= generalized vasoconstriction= decreased CO and renal blood flow option B= short acting acetylcholinesterase inhibitors; diagnosis of myasthenia gravis (i.e., Tensilon test); it reduces heart rate, conduction and contractility option C= low doses= beta-1= increase HR, contractility and CO; intermediate doses= beta-2= vasodilation; high doses= alpha-1= vasoconstriction option D= Cardioselective beta-1 blocker; short duration of action; decreases HR, contractility and conduction without affecting renal blood flow option E= selective alpha-1 agonist= vasoconstriction

44 years man given enalparil for hypertension decreased GFR over the next 7 days of therapy structure responsible for renal response? a)Afferent arterioles b)Distal tubules c)Efferent arterioles d)Proximal tubules e)Vasa recta

answer: efferent arterioles -Enalapril is an ACEI -All ACEIs decrease circulating levels of angiotensin II, a substance that causes 1) systemic vasoconstriction 2) preferential constriction of the glomerular efferent arteriole 3) enhancement of adrenal aldosterone secretion -Reduced AT II= decreased efferent arteriolar resistance and systemic vascular resistance -Selective efferent arteriolar dilation combined with the decreased renal perfusion pressure= reduction in GFR

43 years woman -Occasional chest discomfort (1 year); midline pain (pressure or squeezing= 10-15 mins) sometimes with sweating -ECG= transient ST-segment elevations in anteroseptal leads (during episode of chest pain at night) -Coronary angiography= no atherosclerotic stenosis, but administration of acetylcholine elicits a similar chest pain and ECG changes -Explanation? a)Decreased systemic resistance b)Endothelial dysfunction c)Increased venous return d)Positive inotropic effect e)Reflex tachycardia

answer: endothelial dysfunction -Recurrent chest discomfort + spontaneously resolves within 10-15 mins= vasospastic angina -Cause: coronary endothelial dysfunction and autonomic imbalance= intermittent coronary vasospasm and myocardial ischemia -Affected patients= young (e.g., age< 50) and lack typical risk factors for CAD (e.g., hypertension, diabetes mellitus); however, smoking is a risk factor -Symptoms are triggered by excess vagal tone, and occur most commonly at night when vagal tone is at peak -Diagnosis= ambulatory ECG demonstrating ST-segment elevation during episode of chest pain; coronary angiography= no obstructive atherosclerotic CAD -Both acetylcholine and ergot alkaloids (e.g., dihydroergotamine) provoke symptoms and may also aid in diagnosis -Acetylcholine stimulates endothelial muscarinic receptors to cause vasodilation via increased release NO; however, a deficiency of endothelial NO in affected patients causes increased vagal tone to instead cause vasoconstriction and precipitate vasospastic symptoms -Ergot alkaloids- activate 5-HT2 receptor- vasoconstriction (usually offset by endothelial release of vasodilatory prostaglandins) option A= decreased due to peripheral vasodilation of Ach= lowering BP= reduces coronary artery perfusion pressure option C= peripheral endothelial function is intact in vasospastic angina; Ach causes peripheral vasodilation= decreases venous return (venodilation) option D= Ach has negative inotropic effect, reducing cardiac contractility and myocardial oxygen demand option E= Ach slows down heart rate, counteracting reflex tachycardia caused by vasodilation

53 years man -Follow up visit after acute MI -Medications= metoprolol and low dose aspirin -Quit smoking after MI -Father= hypertension; mother= T2D -Exam: obese male -Normal total cholesterol, low HDL, normal LDL and triglycerides -Best lipid lowering agent for preventing future CVD events? a)Absorption inhibitor b)Cationic exchange resin c)Enzyme inhibitor d)Essential fatty acids e)Pharmacologic vitamin f)Transcription factor ligand

answer: enzyme inhibitor -Patients with low HDL levels are at increased risk of CVD -HDL is involved in reverse cholesterol transport, it helps remove cholesterol from peripheral tissues and transports it to the liver for metabolism à 2 pathways -Direct pathway= HDL delivers cholesterol esters directly to the liver via a scavenger receptor (SCARB1) on hepatocyte cell membrane -Indirect pathway= cholesterol in HDL is transferred to LDL and VLDL by cholesteryl ester transfer protein -Nonpharmacological measures: exercise, weight loss, smoking cessation help raise HDL levels -Medications: 1) medications that raise HDL DOES NOT improve cardiovascular outcomes 2) treatment should be focused on lowering LDL (with HMG-CoA reductase inhibitors= statins)= decrease risk of CVD -Statins are indicated for secondary prevention in all patients with known atherosclerotic CVD, regardless of baseline lipid levels option A= Ezetimibe= inhibits cholesterol absorption in GI; mixed evidence for reduction of risk of CVD; considered in patients who can't take statins option B= cholestyramine; decrease LDL by interfering with enterohepatic circulation of bile acids; can increase triglyceride levels; not proven to prevent cardiac events option D= decrease triglycerides and slightly increase HDL and LDL; do not reduce cardiovascular events option E= raises HDL; but doesn't reduce risk of CVD; adverse effects= hyperglycemia and flushing option F= fibrates activate paroxysmal proliferator-activated receptor alpha, transcription factor that increases lipoprotein lipase activity= decrease triglycerides and raise HDL levels; inferior to statins for reducing risk of CVD; used to prevent pancreatitis in patients with very high triglyceride levels

17 years girl -Episode of syncope (when watching TV sitting); regained consciousness spontaneously after 2 mins -Brief twitching while unconscious -Patient takes antimicrobial agents for recent infection -Uncle= died suddenly 2 years ago -Evaluation= decrease in function mutation of the KCNH2 gene, which codes for voltage-gated potassium channel that is active during repolarization phase of cardiomyocyte AP -Patient takes which medication? a)Acyclovir b)Amoxicillin c)Cephalexin d)Doxycycline e)Erythromycin f)Terbinafine

answer: erythromycin -Mutation= common cause of congenital long QT syndrome -Either autosomal dominant (Romano-Ward syndrome) or autosomal recessive (Jervell and Lange-Nielsen syndrome) -Mutation= impairs function of voltage-gated potassium channels controlling delayed rectifier potassium current that allows repolarization of cardiomyocyte AP -Delayed repolarization= prolonged QT= torsades de pointes (form of polymorphic ventricular tachycardia); can cause syncope or SCD -Most patients with congenital LQTS do not develop TdP unless exposed to external factors (medications= acquired LQTS or exacerbate congenital LQTS) -Macrolide antibiotics (e.g., erythromycin, azithromycin) prolong QT and increase risk of TdP (especially in patients with underlying congenital LQTS) option A= some antiviral medications may prolong QT- including protease inhibitors in treatment of HIV; like saquinavir, atazanavir option F= some antifungals prolong QT; azoles= fluconazole, voriconazole

43 years man -Persistent headaches (3-day history) -History: hypertension -BP (224/115), mildly confused, fundoscopy (bilateral papilledema) -Creatinine slightly high, IV medication initiated that causes arteriolar dilation while also improving renal perfusion and increasing natriuresis -Agent? a)Esmolol b)Fenoldopam c)Hydralazine d)Nitroglycerin e)Phenylephrine

answer: fenoldopam -Severe hypertension + end-organ damage (encephalopathy/confusion, retina/papilledema, acute kidney injury/elevated creatinine)= hypertensive emergency (requires aggressive treatment with immediate but controlled BP reduction to minimize organ damage) -IV agents are preferred in hypertensive emergency due to their rapid action and dose titration ability -Fenoldopam is a short-acting, selective, peripheral D1 agonist with little to no effect on alpha or beta receptors -D1 stimulation= increases cAMP= vasodilation of most arterial beds with corresponding decrease in BP -Renal vasodilation is most prominent= increased renal perfusion, increased urine output and natriuresis (i.e., sodium excretion) -VERY useful in hypertensive emergency with renal insufficiency option A= short-acting, Cardioselective beta-1 antagonist= reduces HR and myocardial contractility; not vasodilator= useful in aortic dissection option C= direct arteriolar vasodilator; however, does not dilate renal arterioles and may decrease renal perfusion and encourage sodium and fluid retention; often used in hypertensive emergency because it is associated with reflex sympathetic activation option D= rapid-acting venodilators that decreases preload and CO; causes minimal arterial dilation and does not improve renal perfusion; mostly used to decrease myocardial oxygen demand in acute coronary syndrome option E= alpha-adrenergic agonist, causes increase in SVR due arterial vasoconstriction; used in hypotension or shock; contraindicated in hypertensive emergency

45 years man -Started on medication for paroxysmal atrial fibrillation -Stress test (9 mins)= reaches 98% of age-predicted maximal heart rate without chest pain or ST segment changes -Pre-test: QRS complex (95msec= normal is 80-120msec) and corrected QT interval (410msec; normal <440) -During maximal HR= QRS complex (125msec) and QTc (400msec) -Based on results, which medication was used? a)Atenolol b)Digoxin c)Dofetilide d)Flecainide e)Verapamil

answer: flecainide -QRS complex duration is typically slightly reduced during exercise in response to increase in cardiac conduction velocity that accompanies faster HR -QRS here normal during rest, increased during stress test= drug used to treat Afib= lengthens QRS duration in rate dependent manner (consistent with drug exhibiting strong use-dependence)= such as flecainide -Flecainide is Class 1C= used to treat SVT (i.e., atrial fibrillation); bind to fast sodium channels responsible for phase 0 depolarization (blocking inward Na current)= prolongs QRS duration with little effect on total QT interval duration -Class 1C= slowest to dissociate from sodium channel (do so primarily during diastole)= use-dependence (sodium blocking effects intensify as HR increases due to less time for medication to dissociate from receptor) option A= Cardioselective beta blocker affect PR interval; during stress test= limit maximal HR achieved option B= can cause false ST segment depression during stress test, and lower maximal achievable HR option C= class III; block K channels= QT prolongation; demonstrate reverse use-dependence (the slower the HR, the more QT interval is prolonged option E= class IV; slows SA and AV conduction; also increases coronary blood flow and reduces myocardial oxygen demand, which can mask ischemia during stress test

23 years man -Exertional chest discomfort with shortness of breath (5-10 mins) -Uncle: died suddenly at 35 years -Apical impulse= strong and sustained -Soft crescendo-decrescendo systolic murmur at apex and left sternal border while supine that becomes quite pronounced when he stands up -Medications to avoid? a)Amiodarone b)Disopyramide c)Isosorbide dinitrate d)Metoprolol e)Verapamil

answer: isosorbide dinitrate -Presentation + family history of SCD + systolic murmur that accentuates with standing from supine= hypertrophic cardiomyopathy Patients with HCM have dynamic LV outflow tract obstruction that worsens with decreased LV volume (decreased preload or reduced SVR); medications that should be avoided in patients with HCM include: -Vasodilators (e.g., dihydropyridine CCBs, nitroglycerine, ACEIs)= decrease SVR= decreased afterload and lower LV volumes -Diuretics= decrease LV venous filling (preload) and also result in greater outflow obstruction In contrast, negative inotropic agents, such as beta blockers (metoprolol), CCBs (verapamil), and disopyramide reduce LVOT obstruction and are helpful in patients with HCM (choices B, D and E) option A= occasionally used to manage atrial or ventricular arrhythmias in patients with HCM

72 years man -Waxing and waning chest pressure (several hours); occasional similar sensation (5-10 mins) on exertion -History: hypertension and hyperlipidemia (nonadherent to therapy) -Smoker (25 years); BP (211/105) -Medication is administered IV and it doesn't change Stroke volume, but decreases HR and systemic vascular resistance -Medication? a)Hydralazine b)Labetalol c)Metoprolol d)Nitroprusside e)Phentolamine

answer: labetalol -SBP > 180 (or DBP > 120) along with evidence of associated end-organ damage (e.g., chest pressure, ST segment changes on ECG)= hypertensive emergency -Patient= underlying CAD and experiencing myocardial ischemia due to increased myocardial oxygen demand (caused by high BP and increased LV afterload) -Management: labetalol (nonselective vasodilatory beta blocker; blocks beta-1, beta-2, alpha-1) -alpha-1 block (outweighs beta-2 blockade)= peripheral vasodilation of both venules and arterioles -Venular dilation= decreases venous return -Arteriolar dilation= decreases SVR -Thus, vasodilation is balanced= stroke volume unchanged -Normally, reduced BP stimulates reflex tachycardia, but strong beta-1 blockade overrides this response= decreased heart rate option A= arteriolar vasodilator=reduce SVR= reflex tachycardia; stroke volume increased option C= Cardioselective beta blocker; only blocks beta-1; mild decrease in BP due to reduced contractility and heart rate, that reduces SV and CO option D= arterial and venous vasodilation=reduce SVR= reflex tachycardia; stroke volume unchanged option E= nonselective alpha blocker=vasodilation= reduce SVR= reflex tachycardia; stroke volume unchanged

69 years man -Chest tightness on exertion (6 months), radiates to throat (lasts 5-10 mins) -History: hyperlipidemia and T2D -Father: coronary artery bypass at 60; mother= stroke -Medication= metabolized to S-nitrosothiols in vascular smooth muscle cells= reports rapid relief -Where does the drug act? a)Cardiac muscle b)Large arteries c)Large veins d)Precapillary sphincters e)Small arteries and arterioles

answer: large veins -Patient has stable angina and experienced relief after nitrate -Nitrates (e.g., nitroglycerine, isosorbide mononitrate, isosorbide dinitrate) are metabolized to nitric oxide and S-nitrosothiols via mitochondrial aldehyde dehydrogenase within vascular SMCs= these activate guanylate cyclase and promote conversion of GTP to cGMP= leads to dephosphorylation of myosin light chains and causes vascular smooth muscle relaxation -Nitrates act predominantly on large veins= venodilation= reduced preload (venous pooling)= reduction in LV EDV and EDP= decrease in LV wall stress= decreased myocardial oxygen demand= relief in angina option A= beta blockers and Nondihydropyridine CCBs are negative inotropic and chronotropic agents; reduce contractility and HR= decrease oxygen demand option B= Sodium nitroprusside generates NO while in bloodstream (not in SMCs); causes balanced vasodilation of both large arteries and veins; used in hypertensive emergency option E= dihydropyridine CCBs (e.g., amlodipine, nifedipine and nicardipine) block calcium channels in vascular smooth muscles to trigger vasodilation; peripheral vasodilation= reduce afterload and cardiac work and oxygen demand; coronary vasodilation= increases myocardial oxygen supply; hydralazine also acts on small arterioles= direct vasodilator

-Age: 71 years woman -History: systolic heart failure and hospitalized 2 weeks ago for exacerbation (treated with IV diuretics) -Discharged with oral diuretics= no shortness of breath anymore, but now muscle weakness and cramping -Exam: decreased muscle strength in lower extremities -Lab= hypokalemia -Another diuretic added to prevent this adverse, acts on? a) proximal convoluted tubule b) late proximal tubule c) descending loop of Henle d) ascending loop of Henle e) early distal convoluted tubule f) late distal tubule and collecting duct

answer: late distal tubule and collecting duct -Symptomatic hypokalemia= most likely due to loop diuretic therapy (e.g., furosemide) used to reduce symptoms of heart failure (e.g., dyspnea, edema) -Can add potassium sparing diuretics (cause hyperkalemia)= balance out loop diuretic hypokalemia -ALL diuretics except for potassium sparing class cause potassium loss by increasing sodium delivery to late distal tubule and cortical collecting duct, where aldosterone-induced sodium reabsorption occurs at the expense of potassium -Aldosterone antagonists: spironolactone, eplerenone -ENaC inhibitors: amiloride, triamterene -Spironolactone and eplerenone are preferred in heart failure, because they can prevent deleterious effects of aldosterone induced cardiac remodeling option A= carbonic anhydrases inhibitors (e.g., acetazolamide, dorzolamide) block sodium and bicarbonate reabsorption in PCT; usually used for altitude sickness or glaucoma option B and C= osmotic diuretics (e.g., mannitol) prevent sodium and water reabsorption in late proximal tubule and descending limb; usually used for cerebral edema with increased ICP option D= Loop diuretics (e.g., furosemide, bumetanide, torsemide) ; inhibit Na/K/2Cl; treat heart failure option E= thiazides (e.g., hydrochlorothiazide, chlorthalidone); inhibit Na/Cl; used in hypertension and sometimes added to loop diuretics in patients with refractory edema, but worsen hypokalemia

49 years woman -Squeezing chest pain and profuse sweating (2 hours) -History: diet controlled T2D -ECG: ST elevations in leads I, aVL, and V1-V4 -Catheterization lab: complete occlusion of the LAD artery= opened with PCI with stenting= afterward she experiences recurrent and sustained episodes of ventricular arrhythmia -Antiarrhythmic (given): binds to rapidly depolarizing and ischemic ventricular myocardial fibers and has minimal effect on normal ventricular myocardium -Agent used? a)Adenosine b)Digoxin c)Diltiazem d)Ibutilide e)Lidocaine f)Metoprolol g)Procainamide

answer: lidocaine -Anterolateral STEMI with successful reperfusion following PCI -Ventricular arrhythmias (premature beats, ventricular tachycardia/fibrillation) are common in the first 24-48 hours after MI (may be treated with antiarrhythmics; e.g., amiodarone, lidocaine) -Class IB drugs (lidocaine) are weakest sodium channel blockers (dissociate the fastest) compared to other class I antiarrhythmics= predominantly bind sodium channels in inactivated state, and dissociation occurs rapidly (negligible effect on QRS duration in normal cardiac tissue) -Ischemic myocardium= higher than normal (less negative) RMP= delays voltage-dependent recovery of Na channels from inactivated to resting state (increased binding of Class IB)= highly efficacious in inhibiting ischemia-induced ventricular arrhythmias option A= transient conduction delay through the AV node= acute treatment of PSVT, not indicated in ventricular arrhythmias option B= enhances vagal tone= leads to increased ERP and decreased conduction through AV node= useful in SVT like atrial fibrillation option C= class IV; slows sinus rate, prolongs conduction through AV node, depress myocardial contractility; used primarily as rate control agent in management of atrial tachyarrhythmias option D= class III; potassium channel blocker; used for acute termination of atrial fibrillation and flutter; prolongs QT and can cause TdP option F= selective beta-1 antagonist; class II; negative inotropic and chronotropic agent; used in patients with acute or recent MI; they reduce risk of ventricular arrhythmias by decreasing automaticity and size of ischemic myocardium by decreasing myocardial oxygen demand option G= class IA; bind sodium channels in open state and more specific for suppressing arrhythmias arising from areas of normal automaticity; have some potassium channel blocking activity= leading to prolongation of APD and increased risk of ventricular arrhythmias

58 years man -Progressive fatigue, dyspnea on exertion, orthopnea -History: hypertension, hyperlipidemia, MI (2 years ago) -BP is high, bilateral lung crackles and lower extremity edema -Point of maximal impulse displaced towards axilla, S3 -Long-term use of which will reduce mortality? a)Amlodipine b)Digoxin c)Diltiazem d)Furosemide e)Hydrochlorothiazide f)Lisinopril g)Triamterene

answer: lisinopril -Patient with previous MI= most likely developed chronic heart failure with reduced ejection fraction (HFrEF) -Appropriate pharmacologic management to reduce mortality (mortality benefit= reduction in cardiac remodeling) -Neurohumoral pathways involving NE, AT II, and aldosterone all have a demonstrated role in cardiac remodeling -in particular, elevated AT II and aldosterone levels increase pathologic cardiomyocyte hypertrophy and stimulate collagen deposition by fibroblasts -Accordingly, ACEIs, ARBs, aldosterone antagonists reduce mortality in patients with HFrEF via reduction of AT II and aldosterone mediated cardiac remodeling -Beta blockers also reduce mortality in patients with chronic HFrEF= similar neurohumoral mechanisms -Potentiation of the natriuretic peptide neurohumoral pathway (e.g., with sacubitril-valsartan) also has mortality benefit in these patients option A= improves cardiac remodeling due to LV pressure overload due to hypertension option B= improve symptoms and reduce hospitalization only option C= depress cardiac contractility and usually contraindicated in patients with HFrEF option D and E= reduce cardiac preload= for symptom control option G= they block ENaC not aldosterone; minimal benefit on mortality in HFrEF

60 years man -Chest pain; intermittent, squeezing, substernal pain (3 days); now pain is sustained -History: moderate COPD, recent admission for exacerbation -High BP; ECG: 2-mm anterior ST elevation -Cardiac troponin I levels are elevated -Treatment? a)Ibutilide b)Metoprolol c)Nadolol d)Propranolol e)Sotalol

answer: metoprolol -Clinical presentation= acute MI -Beta blockers are used in acute MI to reduce myocardial oxygen demand by lowering HR, contractility and afterload (reduce short term morbidity, minimize infarct size and improve long term survival) -Contraindications: bradycardia, heart block, hypotension, overt heart failure (e.g., pulmonary edema) -Also, non-Cardioselective beta blockers (e.g., propranolol, nadolol) can trigger bronchospasm in patients with underlying obstructive lung disease (asthma, COPD) due to beta-2 receptor blockade (choice C and D should be avoided) -Cardioselective beta blockers act on beta-1 only (e.g., metoprolol, atenolol, bisoprolol, nebivolol) are safe in patients with obstructive lung diseases -Combined beta and alpha receptor blockers (e.g., carvedilol, labetalol) are well tolerated and are safe too option A= class III antiarrhythmic; used for acute termination of atrial flutter or atrial fibrillation; can cause QT prolongation option E= class III antiarrhythmic with beta blocker properties; atrial and ventricular arrhythmias; can prolong QT

72 years man -Severe substernal chest pain, shortness of breath (several hours); pain started when working in garden -History: diet-controlled diabetes and hypertension -Exam: bilateral crackles and S3 -ECG: ST elevation in multiple lead -Diagnosis: acute myocardial infarction complicated by acute severe heart failure -Started on dobutamine infusion; it increases? a)Cardiac diastolic filling time b)Myocardial oxygen consumption c)Peripheral vascular resistance d)Pulmonary capillary wedge pressure e)Right ventricular end diastolic pressure

answer: myocardial oxygen consumption -Dobutamine is beta agonist (beta-1 receptors mainly; weaker beta-2) and minimal activity on alpha-1 -Use: management of refractory heart failure associated with severe LV systolic dysfunction and cardiogenic shock Stimulating beta receptors= increased cAMP in target cells: -Positive inotropy and chronotropy: increased cardiac contractility (potent effect) and heart rate (weaker effect), leading to increased CO (improved end-organ perfusion) and decreased LV filling pressures (improves pulmonary congestion) -Mild vasodilation: decreased systemic vascular resistance that often causes a slight reduction in BP Strong inotropic effect of dobutamine= increases myocardial oxygen consumption (can exacerbate ischemia)= shouldn't be used routinely in patients with decompensated heart failure; however, in patients with cardiogenic shock, this drawback is outweighed option A= weak chronotropic agent= increase HR= decrease in cardiac diastolic filling time option C= beta-2 effect stronger than alpha-1 in dobutamine= mild vasodilation; decrease in SVR and slight reduction in BP option D and E= decreases it due to increased contractility and output)

62 years man -Follow up of hypertension (treatment with several medications) -Patient had to discontinue several medications (side effects= palpitations, dizziness, headaches) -Currently he takes ramipril and chlorthalidone and is tolerating them well -BP is high; ECG= bradycardia with PR prolongation -Medication to lower BP without worsening ECG abnormalities? a)Diltiazem b)Metoprolol c)Nifedipine d)Sotalol e)Verapamil

answer: nifedipine -CCBs are used in treatment of hypertension, angina, and certain arrhythmias Inhibit L-type Ca2+ channel on vascular SMCs and cardiac cells and divided into two groups -Dihydropyridines (nifedipine, amlodipine, felodipine)= primarily affect arterial smooth muscle= vasodilation with little effect on heart -Nondihydropyridines (verapamil, diltiazem) affect the myocardium, slowing HR and reducing contractility ECG= bradycardia with first degree AV block (PR> 200msec) Nifedipine is the most appropriate to manage hypertension (no cardiac side effects)= cause reflex tachycardia in response to vasodilation

56 years man -Palpitations and lightheadedness (2 hours) -History: hypertension, alcohol daily -BP (132/76) and high pulse (irregular) -ECG= atrial fibrillation -Cardioversion then given amiodarone (to maintain normal sinus rhythm) -Changes due to medication? a)Sinus rate: decreased PR interval: prolonged QRS duration: no change QT interval: no change b)Sinus rate: decreased PR interval: prolonged QRS duration: prolonged QT interval: prolonged c)Sinus rate: no change PR interval: no change QRS duration: no change QT interval: no change d)Sinus rate: no change PR interval: no change QRS duration: prolonged QT interval: no change e)Sinus rate: no change PR interval: no change QRS duration: prolonged QT interval: prolonged

answer: option B (Sinus rate: decreased PR interval: prolonged QRS duration: prolonged QT interval: prolonged) -Amiodarone= wide-ranging antiarrhythmic effects (mimics class I, II, III, IV) but classified under class III -Primarily inhibits rapid component of delayed rectifier potassium current, which is responsible for ventricular repolarization= prolongs ERP, suppresses electrical foci that stimulate atrial and ventricular arrhythmias -Prolongs QT (increases risk of TdP, BUT much lower risk than other class III drugs) Amiodarone has the following effects: -Class I effect: somewhat inhibits fast Na channels; responsible for rapid depolarization= prolong QRS complex -Class II and IV effects: has beta blocking properties that inhibit SNS stimulation of conduction system; also inhibits L-type Ca2+ channels= responsible for depolarization of SA node and AV node= prolong refractory period= suppress conduction system to suppress arrhythmogenic foci and in doing so= decreases sinus rate and prolongs PR interval option A= class II and IV option C= class IB option D= class IC option E= class IA

64 years man -Long-standing hypertension, dry cough, shortness of breath -Unable to sleep in flat position for past 2 days -BP (192/102), S4 and bibasilar crackles -Started on IV nitroglycerine infusion= symptomatic relief -Physiologic changes? a)Left ventricular end-diastolic pressure: no effect; peripheral venous capacitance: increased; systemic vascular resistance: decreased b)Left ventricular end-diastolic pressure: decreased; peripheral venous capacitance: increased; systemic vascular resistance: no effect c)Left ventricular end-diastolic pressure: increased; peripheral venous capacitance: decreased; systemic vascular resistance: increased d)Left ventricular end-diastolic pressure: decreased; peripheral venous capacitance: increased; systemic vascular resistance: decreased e)Left ventricular end-diastolic pressure: decreased; peripheral venous capacitance: decreased; systemic vascular resistance: decreased f)Left ventricular end-diastolic pressure: increased; peripheral venous capacitance: no effect; systemic vascular resistance: decreased g)Left ventricular end-diastolic pressure: increased; peripheral venous capacitance: increased; systemic vascular resistance: increased

answer: option D (Left ventricular end-diastolic pressure: decreased; peripheral venous capacitance: increased; systemic vascular resistance: decreased) -presentation: acute pulmonary edema due to severely elevated BP (hypertensive emergency) -IV vasodilators (e.g., nitroglycerine, sodium nitroprusside) are often used in management of acute heart failure due to severe hypertension -Nitrates are primary venodilators that increase peripheral venous capacitance= reduce cardiac preload= reduce LV EDP and EDV (symptom relief in pulmonary edema) -Lower LVEDP also leads to a reduction in LV systolic wall stress and decrease in myocardial oxygen demand (relief of angina) -Modest effect on arteriolar dilation (higher doses can lead to significant drop in systemic BP, thereby reducing cardiac afterload, reducing LV wall stress and causing further decrease in myocardial oxygen demand)

60 years man -Intermittent, mild, generalized headache -High BP= started on lisinopril -Dry cough after starting drug; drug is stopped, and losartan is started -Effects of losartan? a)Renin: decreased; AT I: decreased; AT II: decreased; Aldosterone: decreased; Bradykinin: no change b)Renin: increased; AT I: decreased; AT II: decreased; Aldosterone: decreased; Bradykinin: no change c)Renin: increased; AT I: increased; AT II: decreased; Aldosterone: decreased; Bradykinin: increased d)Renin: increased; AT I: increased; AT II: increased; Aldosterone: decreased; Bradykinin: no change e)Renin: increased; AT I: increased; AT II: increased; Aldosterone: increased; Bradykinin: no change

answer: option D (Renin: increased; AT I: increased; AT II: increased; Aldosterone: decreased; Bradykinin: no change) -Renin release= stimulated by decreased sodium delivery to distal tubule, low BP (e.g., low intravascular volume), beta-1 sympathetic activity -renin converts angiotensinogen to AT I, which is then converted to AT II by ACE (also degrades bradykinin) -AT II stimulates type 1 angiotensin II receptors found on vascular smooth muscle (causing vasoconstriction) and adrenal cortex (stimulates aldosterone secretion= water and sodium reabsorption= raise BP= inhibiting further renin release -ARBs (angiotensin II receptor blockers)= block effects of AT II= vascular smooth muscle relaxation and decreased aldosterone= reduce BP= stimulates renin, increases AT I and AT II (bradykinin levels are unaffected) -Choice C= ACEIs (coughing b/c bradykinin increases prostaglandin synthesis= bronchial irritation) -Choice A= beta-1 blockers -Choice B= direct renin inhibitors -Choice E= aldosterone antagonists (e.g., spironolactone)

-Series of experiments on hypophysectomized animals to investigate effects of cortisol on vascular reactivity -Initiate tests= administration of cortisol alone= no effect -Next= norepinephrine with and without cortisol -Results: norepinephrine + cortisol increases vascular response compared to norepinephrine alone -Principle of cortisol effect? a)Additive effect b)Alteration in metabolism c)Permissiveness d)Synergistic effect e)Tachyphylaxis

answer: permissiveness -Cortisol has no direct vasoactive properties, it augments the vasoconstrictive effects of catecholamines and angiotensin II -Above graph= limited degree of vasoconstriction (NE alone); that increased following pretreatment with cortisol= permissiveness -Permissiveness= when hormone has no direct effect on a physiologic process but allows another hormone to exert its maximal effect on that process -Cortisol= potentiating effect of vasoconstriction via upregulation of alpha-1 on vascular SMCs -In adrenal insufficiency= low cortisol levels can contribute to hypotensive crisis by decreasing vascular responsiveness to NE and angiotensin II -Cortisol also has a permissive effect on catecholamine-mediated bronchodilation and glucagon mediated release of glucose from liver choice A= 2 drugs with similar actions can have synergistic or additive effect; when the combined effect of 2 drugs is equal to the sum of their individual effects= additive response (e.g., 1+1=2) choice B= can change pharmacodynamic and kinetic properties; e.g., certain drugs like amiodarone can inhibit P450 system; decreasing metabolism of other drugs option D= 2 drugs with similar actions can have synergistic or additive effect; when the combined effect exceeds the sum of the individual drugs effects= synergistic (e.g., 1+1>2) option E= describes drug responsiveness in short period following one or more doses (i.e., rapidly developing tolerance)

-age: 37 years man -Intermittent palpitations -ECG= atrial fibrillation with rapid ventricular response -Medication: effective against atrial and ventricular arrhythmias (causes QRS prolongation with only minimal increase in QT interval duration) -Affects which part of action potential? a) phase-0 b) phase-1 c) phase-2 d) phase-3 e) phase-4

answer: phase-0 -Most likely treated with Class IC antiarrhythmic agent, which typically causes QRS prolongation with little effect on QT interval -QRS complex represents rapid ventricular depolarization and corresponds to phase 0 of ventricular myocyte action potential -QT interval= represents both ventricular depolarization and repolarization (duration of AP); QRS only short time= QT interval is mostly a measure of ventricular repolarization -Ventricular depolarization= Na channels (Class I block them)= prolong QRS -Class IA (e.g., procainamide)= increase QRS duration and moderate potassium channel blocking= QT prolongation -Class 1B (e.g., lidocaine)= no significant effect on QRS duration and do not prolong QT (rapid dissociation from receptor) -Class IC (e.g., flecainide, propafenone)=prolong QRS, little effect on QT

45 years man -Recurrent palpitations, chest discomfort, shortness of breath -Year ago= paroxysmal atrial fibrillation (treated with rate control with beta-blocker) -History= hypertension and obesity -Echo= LA enlargement, normal LV ejection fraction -24-hour Holter monitoring= bursts of atrial fibrillation associated with patient's symptoms -Initiated on Dofetilide to maintain normal sinus rhythm; effect of medication on AP? a) phase-0 b) phase-1 c) phase-2 d) phase-3 e) phase-4

answer: phase-3 -Phase 0 (rapid depolarization)= sodium channels (class I antiarrhythmics; e.g., procainamide, flecainide, lidocaine) -Phase 1 (early repolarization)= transient outward potassium current (not modulated by antiarrhythmics) -Phase 2 (plateau)= balance of "late" inward calcium current and potassium efflux= corresponds to phase 0 in pacemakers= CCBs (class IV) block L-type Ca channels= slow sinus rate, prolong conduction through AV and depress myocardial contractility -Phase 3 (late repolarization)= opening of potassium channels (delayed rectifiers= IKr and IKs)= Class III (amiodarone, sotalol, Dofetilide)= prolong repolarization and APD= prolong QT interval= these medications are used for maintenance of sinus rhythm in patients with paroxysmal atrial fibrillation -Phase 4 (recovery)= RMP (inward potassium rectifying current)= no action of antiarrhythmics

-Experiment: investigating vasoconstriction of arterial wall, 2 samples of isolated porcine arterial vessels studied -Both infused with NE and vascular tone is monitored -Second vessel= pretreated with drug A before NE infusion -Results are shown; drug A is most likely? (learn don't answer) a)Atropine b)Labetalol c)Phenoxybenzamine d)Phentolamine e)Propranolol

answer: phenoxybenzamine -alpha-1= vasoconstriction; beta-2= vasodilation -NE= alpha-1 and beta-1 agonist; less action on beta-2= vasoconstrictor -Test= efficacy of drug A in blocking alpha-1 constriction -Drug A= decreased Vmax of NE without significant change in affinity (Km) of NE on alpha-1= drug A is either noncompetitive or irreversible antagonist of alpha-1 receptors -Phenoxybenzamine is an irreversible alpha-1, alpha-2 receptor antagonist (primarily used in treatment of pheochromocytoma which causes vasoconstriction due to NE and Epi release) option A= competitive muscarinic antagonist option B= reversible, competitive antagonist of alpha-1 and beta-1 receptors with partial beta-2 agonist activity= treats hypertension); high doses of NE can overcome alpha inhibition option D= a)reversible, competitive alpha-adrenergic antagonist= treats catecholamine induced hypertensive crisis (e.g., pheochromocytoma, MAOI toxicity, cocaine intoxication); high doses of NE can overcome alpha- inhibition option E= nonspecific beta-adrenergic antagonist; potentiates vasoconstrictive effects of NE due to unopposed alpha stimulation

67 years man -Right lower lobe pneumonia and subsequently develops hypotension and lactic acidosis -Started on norepinephrine IV drip; few hours later= antecubital vein (used for infusion)= blanches and tissues surrounding IV site become cold, hard, and pale -Local injection of affected tissues with will cause greatest benefit? a)Calcium gluconate b)Phentolamine c)Heparin d)Isoproterenol e)Lidocaine

answer: phentolamine -Development of venous blanching along with induration and pallor of tissues surrounding NE infusion site= signs of NE extravasation -NE leak causes intense alpha-1 mediated vasoconstriction which can lead to local tissue necrosis -Such necrosis can be prevented by infiltration (using a syringe with a fine hypodermic needle) throughout the affected area with phentolamine, an alpha receptor blocker= vasodilation (reverse vasoconstrictive effect) -This antidote must be given within 12 hours of extravasation to be effective option A= administered for severe hypocalcemia and hyperkalemia; cardioprotective agent that prevents arrhythmias option C= indicated if there is an arterial thrombosis (cold, pulseless, painful limb); no risk factors of arterial thrombosis here, like atrial fibrillation and arterial catheterization option D= can mediate vasodilation via beta-2 agonism; in striated muscle, renal, and mesenteric vascular beds= decreased peripheral vascular resistance and increased CO; subcutaneous vessels have low levels of beta-2 receptors; alpha blocker more effective option E= can relax vascular smooth muscle via blockade of Na current mediated AP; in this case extravasation causes vasoconstriction; nerve blockade wouldn't reverse this ischemia

65 years Eastern European -Low grade fever, multiple joint pains, well-demarcated erythematous rash on face and trunk -Denies hair loss, mucosal ulceration, or photosensitivity -Serum test= positive ANA -History= CAD, CHF, arrhythmias -Should be questioned about intake of? a)Amiodarone b)Propranolol c)Lidocaine d)Procainamide e)Verapamil f)Adenosine

answer: procainamide -Drug induced lupus erythematosus (DILE) should be considered in patients with symptoms of SLE -High risk drugs: hydralazine, procainamide, isoniazid -Procainamide is metabolized via hepatic acetylation= individuals who are slow acetylators are at greatest risk -In 20% of patients who take procainamide -Autoantibodies: antihistone antibodies (very common; but only 50% in SLE); anti-dsDNA (80% in SLE, rare in DILE) -ANA positive in both option A= thyroid dysfunction, lung fibrosis, liver toxicity, blue-grey discoloration of skin option B= negative inotropy and bronchoconstriction in asthmatics and COPD patients option C= class 1b; tremor, drowsiness, change in mental status option E= calcium channel blocker and class 4 antiarrhythmic; negative inotropy, constipation, and gingival hyperplasia option F= vasodilator in stress tests and fast acting antiarrhythmic for stopping acute supraventricular tachycardias; very short half-life; chest burning, flushing and transient hypotension

-Experiment -First group: low-dose epinephrine= DBP decreased and HR increased -Second group: pretreated with drug A then infused with low-dose epinephrine= DBP increases and HR decreases (reversed effects) -Drug A? a)Atropine b)Isoproterenol c)Phentolamine d)Phenylephrine e)Propranolol

answer: propranolol -low-dose epinephrine= beta-1= beta-2> alpha-1 -Stimulation of beta-1= increased HR, contractility (increase SBP and CO) -Stimulation of beta-2= vasodilation= decreased DBP -Simulation alpha-1= vasoconstriction= increased DBP -Nonselective beta antagonist= inhibit beta-1 and beta-2= decrease HR and increase DBP= propranolol= only alpha-1 effect left= vasoconstriction= increase DBP and reflex bradycardia option A= atropine inhibits cholinergic receptors (M2 in heart); increases HR option B= stimulates beta-1 and 2= decrease DBP and increase HR option C= nonspecific alpha blocker= fall in DBP and reflex tachycardia option D= selective alpha agonist= elevated BP, reflex bradycardia

6 days boy -Heart murmur; born at 27 weeks of gestation and intubated (surfactant deficiency) -Low birth weight, low BP, and high pulse -Continuous murmur with systolic attenuation is best heard at left heart border -Bounding femoral and palmar pulses -Elevated levels of contributes to condition? a)endothelin-1 b)Leukotriene D4 c)Prostaglandin E2 d)Thromboxane A2 e)Vasopressin

answer: prostaglandin E2 -Preterm + continuous murmur+ bounding pulses + wide pulse pressure= patent ductus arteriosus -Patent pulses ductus arteriosus= connection pulmonary artery trunk to proximal descending aorta= shunts systemic blood into pulmonary artery= eventually pulmonary artery pressure is higher than systemic= right to left shunting (Eisenmenger) -Cause: high prostaglandin E2 (produced by placenta) and relative hypoxemia of fetal circulation -After birth= closure= loss of placental PGE2 and vasoconstrictive effects of increased systemic oxygen levels (e.g., increased endothelin-1 production)= choice A -Can eventually lead to heart failure or progressive pulmonary hypertension -NSAIDS (e.g., indomethacin, ibuprofen)= help with PDA closure by inhibiting synthesis of PGE2 option B= treatment with NSAIDs= increase leukotriene synthesis; facilitates closure of PDA due to vasoconstrictive effects option D= increases platelet production and aggregation; antithrombotic effect of aspirin= decreased TXA2 option E= promotes water reabsorption in kidneys and causes arteriolar constriction

25 years man -Shortness of breath (even on moderate exertion) -Exam: faint systolic murmur at left sternal border in the supine position that increases to 3/6 in intensity with Valsalva maneuver -Echocardiogram= interventricular septal hypertrophy and increased LV mass -Started on: high dose beta blocker therapy -4 weeks later= improvement in shortness of breath -Exam: no murmurs supine or with Valsalva -Why do symptoms improve? a)Dilation of epicardial coronary arteries ( b)Increase in left ventricular outflow tract flow velocity c)Reduction in left ventricular contractility d)Reduction in left ventricular mass e)Reduction in left ventricular preload

answer: reduction in left ventricular contractility -Classic signs of hypertrophic cardiomyopathy (HCM) -Many patients with HCM have LV outflow tract obstruction that can lead to symptoms of poor CO (e.g., dyspnea, syncope) -Outflow tract obstruction worsens with decreased LV blood volume (during exercise due to increased LV filling time and increased LV contractility) -Exam: systolic murmur that increases in intensity with maneuvers that decrease LV blood volume (e.g., Valsalva and abrupt standing) In patients with HCM and symptomatic LV outflow obstruction= beta blockers improve symptoms via an increase in LV volume that reduces LV outflow obstruction and improve cardiac output; 2 mechanisms: -Reduce heart rate, which lengthens diastolic filling time and increases amount of blood that can enter heart during each beat (increased EDV) -Reduce LV contractility, which reduces amount of blood ejected during systole (increased ESV) option A= some patients with HCM may have anginal symptoms due to inadequate blood supply to increased myocardial mass; beta blockers improve these symptoms by decreasing LV wall stress and myocardial oxygen demand, and improving coronary artery perfusion gradient; only minimal effect on dilation of coronaries option B= beta blockers increase cross sectional area of LV outflow tract= decrease flow velocity option D= long term effect; evident after 4 weeks of beta blockers option E= beta blockers increase LV preload to reduced LV outflow tract obstruction

79 years man -Confusion (2 days), nausea, decreased appetite, poor oral intake (weeks) -Everything around him has a "yellow tint" -History: long-standing heart failure with reduced ejection fraction and atrial fibrillation -Medications: metoprolol, digoxin, lisinopril, and apixaban -Age-related changes in which of the following factors accounts for symptoms? a)Intestinal absorption b)Liver enzyme activity c)Muscle mass d)Plasma protein binding e)Renal clearance

answer: renal clearance -digoxin= used to treat atrial fibrillation and systolic heart failure -Digoxin toxicity= visual changes (particularly yellow tinting of objects) and GI disturbances -Digoxin is renally cleared and has narrow therapeutic window, so toxicity frequently results from alterations in kidney function -Over time (increased age)= renal function is decreased= reduce doses of renally cleared medications option C= primary site of digoxin storage= decreases with age; but renal insufficiency is more direct effect option D= 1/3 of digoxin binds proteins; but kidney function is more important

54 years man -Atrial fibrillation with rapid ventricular response -Symptoms began with palpitations, lightheadedness and chest tightness (similar episodes before) -History: mitral valve repair 3 years ago for severe symptomatic mitral regurgitation -Cardioversion + considered for amiodarone therapy for future episodes -Test before initiating amiodarone? a)24-hour urinary cortisol b)24-hour urinary metanephrines c)Oral glucose tolerance test d)Serum prolactin e)Serum testosterone f)Serum TSH

answer: serum TSH -Class III used to suppress life-threatening rhythm disturbances -it is 40% iodine by weight, amIODarone can cause a number of alterations in thyroid function -it can cause hypothyroidism due to decreased production of thyroid hormone -Individuals with pre-existing autoimmune thyroid disease are at greatest risk and so should be screened for subclinical hypothyroidism with a serum TSH assay prior to initiating therapy -Amiodarone can also cause hyperthyroidism due to increased hormone synthesis or destructive thyroiditis with release of preformed thyroid hormone option C= used to screen for diabetes mellitus, primarily in pregnant women option D= prolactin is regulated by inhibitory effects of dopamine from the hypothalamus; release is stimulated to a small degree by TRH and mildly affected by amiodarone- not clinically significant option E= amiodarone can cause erectile dysfunction, which may be related to its alpha and beta blocking effect

46 years man -Resistant hypertension -New long-acting medication that causes selective direct relaxation of smooth muscle of arterioles but doesn't not affect veins -Adverse effect? a)Angioedema b)Bradycardia c)Cold extremities d)Decreased cardiac output e)Persistent cough f)Sodium and fluid retention g)Transient hypertension

answer: sodium and fluid retention -Selective arteriolar vasodilators (e.g., hydralazine, minoxidil) lower BP by reducing SVR= effect limited by reflex sympathetic activation -This leads to 1) increased HR, contractility and CO (choices B and D) -2) Sympathetic stimulation of RAAS results in sodium and fluid retention with peripheral edema -These effects offset much of the BP lowering effect (limit long-term efficacy) -They are used acutely for patients with severely elevated BP; they can also be given in combination with sympatholytics and diuretics to mitigate the side effects and provide synergistic BP lowering in patients with resistant hypertension option A and E= ACEIs option C= beta blockers; decrease CO= peripheral vasoconstriction option G= selective arteriolar vasodilators do not raise BP unless stopped abruptly

50 years man -Recurrent episodes of paroxysmal atrial fibrillation (palpitations and chest pressure) -Medication= 2 weeks later= lightheadedness, weakness, presyncope -ECG= sinus bradycardia and QTc prolongation -Telemetry= short episode of self-resolved TdP -Medication used? a)Diltiazem b)Metoprolol c)Mexiletine d)Ranolazine e)Sotalol

answer: sotalol -Paroxysmal Afib= treated with rate or rhythm control -Rate control using AV nodal block= beta blockers, CCBs= to prevent rapid ventricular response -Rhythm control= maintain sinus rhythm= sotalol, flecainide, amiodarone -Class III antiarrhythmics (e.g., sotalol, amiodarone, Dofetilide) predominantly block potassium channels and inhibit outward repolarizing currents during phase 3 of AP= increased APD and QT interval prolongation (predisposes to polymorphic ventricular tachycardia (torsades de pointes); least risk with amiodarone option A= class IV; CCB; inhibits phase 2 of cardiomyocyte AP and phase 0 of nodal AP; slows sinus rate, prolongs AV conduction time; depresses myocardial contractility option B= class II; selective beta-1 blocker= negative inotropic and chronotropic option C= class IB; blocks Na channels; inhibits phase 0 of cardiomyocyte AP; occasionally used to maintain sinus rhythm in patients with paroxysmal symptomatic atrial fibrillation option D= antianginal agent and exerts its effect by inhibiting the late phase of inward sodium channels in ischemic cardiac myocytes; it also blocks potassium channels and can cause dose dependent increase in QT interval without risk of TdP

67 years man -Nonischemic cardiomyopathy (follow-up) -Recently hospitalized= acute decompensated heart failure -Symptoms improved with multidrug treatment, but has persistent shortness of breath on mild exertion -History: hypertension and hypercholesteremia -Normal BP and pulse -Third heart sound and mild lower extremity pitting edema -Echocardiogram= LV ejection fraction of 30% -Best diuretic to use? a)Acetazolamide b)Furosemide c)Hydrochlorothiazide d)Mannitol e)Spironolactone f)Triamterene

answer: spironolactone -Mineralocorticoid receptor antagonists (e.g., spironolactone, eplerenone) prevent aldosterone from binding to its receptor in the DCT= increased sodium and water excretion while conserving potassium ions (potassium-sparing diuresis) -These antagonists also block the deleterious effect of aldosterone on the heart, causing regression of myocardial fibrosis and improvement in ventricular remodeling -Reduce morbidity and improve survival in patients with CHF and decreased ejection fraction -Recommended in addition to standard heart failure therapy (ACEIs and beta blockers) -Don't use in patients with hyperkalemia or renal failure; side effects: hyperkalemia and gynecomastia with spironolactone option B= frequently used for treatment of pulmonary congestion and fluid retention in heart failure patients; they improve symptoms significantly; they do not improve survival option D= used to lower ICP and treat acute glaucoma; can cause initial rise in ECF volume= exacerbating heart failure

60 years man -CAD; exertional chest pain (6 months; worst in cold weather) -History: coronary artery bypass graft 3 years ago for progressive angina -History: asthma, benign prostate hyperplasia, peripheral artery disease -Physician adds isosorbide dinitrate, side effect? a)Atrioventricular conduction delay b)Cold extremities c)Constipation d)Joint pains e)Nocturnal wheezing f)Throbbing headaches g)Urinary retention

answer: throbbing headaches -Nitrates are used in chronic stable angina for symptomatic relief -Anti-ischemic effect= systemic vasodilation (predominantly veins) with a decrease in LV EDV and wall stress= decreased myocardial oxygen demand and relief of angina symptoms -Adverse effects: headaches, cutaneous flushing, lightheadedness, hypotension and reflex tachycardia -Should be avoided in patients with hypertrophic cardiomyopathy (due to increased outflow tract obstruction), RV infarction (due to reduction in preload, impairing CO), and those on PDE inhibitors (severe hypotension) option A= CCBs (e.g., verapamil, diltiazem) and beta blockers option B= Raynaud phenomenon; associated with amphetamines, ergotamine and chemotherapeutic agents option C= opioids, iron supplements, verapamil, and anticholinergics option D= thiazides= hyperuricemia and precipitate an acute gout attack option E= asthma or GERD; beta blockers in patients with underlying bronchospastic disease option G= anticholinergic properties in TCAs (e.g., amitriptyline), and antihistamines (e.g., diphenhydramine)

62 years man -Mitral valve replacement (1 month ago) -Re-evaluation for low-grade fevers, malaise, dyspnea -Blood cultures= gram-positive cocci in clusters that are catalase-positive and coagulase-negative -Initial empiric antibiotic therapy? a)Ceftriaxone b)Ciprofloxacin c)Clindamycin d)Nafcillin e)Penicillin G f)Vancomycin

answer: vancomycin -infection: coagulase-negative staphylococci -These organisms produce a polysaccharide slime facilitating prosthetic device adherence (catheters, foreign bodies) and common cause of prosthetic valve endocarditis (PVE) -Most common: staphylococcus epidermidis= if left untreated= intracardiac abscess, prosthetic valve dehiscence, and septic embolization -It is part of microflora= true infection or contamination?= perform several blood cultures ->80% are methicillin resistant= should be treated as methicillin resistant until proven otherwise= vancomycin -Additional antimicrobial agents (e.g., gentamicin and/or rifampin, which kills bacteria on foreign material) are administered in some cases of deep-seated methicillin resistant infections or staphylococcal PVE -If methicillin susceptible is found out later= switch to semi-synthetic beta-lactamase resistant penicillin such as nafcillin or oxacillin (choice D) option A= mechanism of methicillin resistance= mecA chromosomal gene= alterations in penicillin binding protein 2a= resistance to penicillins and cephalosporins option B= fluoroquinolone antibiotic; resistance to it accompanies methicillin resistance option E= very few strains of S. epidermidis are susceptible to penicillin G

45 years man -Shortness of breath, fatigue (progressed last 2 weeks) -Diagnosed with: nonischemic cardiomyopathy 2 years ago and has not been adherent with physician visits and medical regimen -BP is low; uncomfortable when lying flat; elevated JVP; bibasilar crackles, S3, 1+ bilateral lower extremity pitting edema -Milrinone infusion is initiated; response to therapy? a)Angioedema b)Antiarrhythmic action c)Atrioventricular conduction block d)Decreased renal perfusion e)Increased right atrial pressure f)Vasodilation

answer: vasodilation -Milrinone is a selective PDE-3 inhibitor= can be used in patients with refractory heart failure due to LV systolic dysfunction PDE-3 normally responsible for degradation of cAMP to AMP; inhibition= increased cAMP: -In cardiomyocytes= intracellular calcium influx increased= increases contractility (positive inotropy)= increased SV and CO -In vascular SMCs= uptake of calcium by SR is increased, which reduces calcium-myosin light chain kinase interaction= stimulate relaxation and vasodilation; venous vasodilation= reduces preload; arterial vasodilation= reduce afterload= cumulative reduction in cardiac work Vasodilation= can lead to hypotension= here compensated by increase in SV and CO to maintain BP option A= ACEIs option C= CCBs, digoxin, beta blockers option D= increased with milrinone option E= decreased with milrinone


संबंधित स्टडी सेट्स

Microeconomics Exam #1 Chapters: 1 & 3

View Set

Gestalt (5 principles of visual perception)

View Set

Financial Math Semester 2 Exam review

View Set

SEC+ Objectives Practice Q&A: CH5 - Governance, Risk, and Compliance

View Set

Chapter 16: Managing Change and Stress

View Set